YOU ARE DOWNLOADING DOCUMENT

Please tick the box to continue:

Transcript
Page 1: Biostatistics 602 - Statistical Inference Lecture 26 Final ... · Biostatistics 602 - Statistical Inference Lecture 26 Final Exam Review & Practice Problems for the Final Hyun Min

..........

.....

......

.....

.....

.....

......

.....

.....

.....

......

.....

.....

.....

......

.....

......

.....

.....

.

. . . . . . . .Review

. . . .P1

. . . . . .P2

. . . . .P3

. . . . . .P4

.Wrap-up

.

......

Biostatistics 602 - Statistical InferenceLecture 26

Final Exam Review & Practice Problems for the Final

Hyun Min Kang

Apil 23rd, 2013

Hyun Min Kang Biostatistics 602 - Lecture 26 Apil 23rd, 2013 1 / 31

Page 2: Biostatistics 602 - Statistical Inference Lecture 26 Final ... · Biostatistics 602 - Statistical Inference Lecture 26 Final Exam Review & Practice Problems for the Final Hyun Min

..........

.....

......

.....

.....

.....

......

.....

.....

.....

......

.....

.....

.....

......

.....

......

.....

.....

.

. . . . . . . .Review

. . . .P1

. . . . . .P2

. . . . .P3

. . . . . .P4

.Wrap-up

Review of the second half

Rao-Blackwell :

If W(X) is an unbiased estimator of τ(θ),ϕ(T) = E[W(X)|T] is a better unbiased estimator for asufficient statistic.

Uniqueness of MVUE : Theorem 7.3.19 - Best unbiased estimator isunique

MVUE and UE of zeros : Theorem 7.3.20 - Best unbiased estimator isuncorrelated with any unbiased estimators of zero

UMVE by complete sufficient statistics : Theorem 7.3.23 - Any functionof complete sufficient statistic is the best unbiased estimatorfor its expected value

How to get UMVUE Strategies to obtain best unbiased estimators:• Condition a simple unbiased estimator on complete

sufficient statistics• Come up with a function of sufficient statistic whose

expected value is τ(θ).

Hyun Min Kang Biostatistics 602 - Lecture 26 Apil 23rd, 2013 2 / 31

Page 3: Biostatistics 602 - Statistical Inference Lecture 26 Final ... · Biostatistics 602 - Statistical Inference Lecture 26 Final Exam Review & Practice Problems for the Final Hyun Min

..........

.....

......

.....

.....

.....

......

.....

.....

.....

......

.....

.....

.....

......

.....

......

.....

.....

.

. . . . . . . .Review

. . . .P1

. . . . . .P2

. . . . .P3

. . . . . .P4

.Wrap-up

Review of the second half

Rao-Blackwell : If W(X) is an unbiased estimator of τ(θ),ϕ(T) = E[W(X)|T] is a better unbiased estimator for asufficient statistic.

Uniqueness of MVUE :

Theorem 7.3.19 - Best unbiased estimator isunique

MVUE and UE of zeros : Theorem 7.3.20 - Best unbiased estimator isuncorrelated with any unbiased estimators of zero

UMVE by complete sufficient statistics : Theorem 7.3.23 - Any functionof complete sufficient statistic is the best unbiased estimatorfor its expected value

How to get UMVUE Strategies to obtain best unbiased estimators:• Condition a simple unbiased estimator on complete

sufficient statistics• Come up with a function of sufficient statistic whose

expected value is τ(θ).

Hyun Min Kang Biostatistics 602 - Lecture 26 Apil 23rd, 2013 2 / 31

Page 4: Biostatistics 602 - Statistical Inference Lecture 26 Final ... · Biostatistics 602 - Statistical Inference Lecture 26 Final Exam Review & Practice Problems for the Final Hyun Min

..........

.....

......

.....

.....

.....

......

.....

.....

.....

......

.....

.....

.....

......

.....

......

.....

.....

.

. . . . . . . .Review

. . . .P1

. . . . . .P2

. . . . .P3

. . . . . .P4

.Wrap-up

Review of the second half

Rao-Blackwell : If W(X) is an unbiased estimator of τ(θ),ϕ(T) = E[W(X)|T] is a better unbiased estimator for asufficient statistic.

Uniqueness of MVUE : Theorem 7.3.19 - Best unbiased estimator isunique

MVUE and UE of zeros :

Theorem 7.3.20 - Best unbiased estimator isuncorrelated with any unbiased estimators of zero

UMVE by complete sufficient statistics : Theorem 7.3.23 - Any functionof complete sufficient statistic is the best unbiased estimatorfor its expected value

How to get UMVUE Strategies to obtain best unbiased estimators:• Condition a simple unbiased estimator on complete

sufficient statistics• Come up with a function of sufficient statistic whose

expected value is τ(θ).

Hyun Min Kang Biostatistics 602 - Lecture 26 Apil 23rd, 2013 2 / 31

Page 5: Biostatistics 602 - Statistical Inference Lecture 26 Final ... · Biostatistics 602 - Statistical Inference Lecture 26 Final Exam Review & Practice Problems for the Final Hyun Min

..........

.....

......

.....

.....

.....

......

.....

.....

.....

......

.....

.....

.....

......

.....

......

.....

.....

.

. . . . . . . .Review

. . . .P1

. . . . . .P2

. . . . .P3

. . . . . .P4

.Wrap-up

Review of the second half

Rao-Blackwell : If W(X) is an unbiased estimator of τ(θ),ϕ(T) = E[W(X)|T] is a better unbiased estimator for asufficient statistic.

Uniqueness of MVUE : Theorem 7.3.19 - Best unbiased estimator isunique

MVUE and UE of zeros : Theorem 7.3.20 - Best unbiased estimator isuncorrelated with any unbiased estimators of zero

UMVE by complete sufficient statistics :

Theorem 7.3.23 - Any functionof complete sufficient statistic is the best unbiased estimatorfor its expected value

How to get UMVUE Strategies to obtain best unbiased estimators:• Condition a simple unbiased estimator on complete

sufficient statistics• Come up with a function of sufficient statistic whose

expected value is τ(θ).

Hyun Min Kang Biostatistics 602 - Lecture 26 Apil 23rd, 2013 2 / 31

Page 6: Biostatistics 602 - Statistical Inference Lecture 26 Final ... · Biostatistics 602 - Statistical Inference Lecture 26 Final Exam Review & Practice Problems for the Final Hyun Min

..........

.....

......

.....

.....

.....

......

.....

.....

.....

......

.....

.....

.....

......

.....

......

.....

.....

.

. . . . . . . .Review

. . . .P1

. . . . . .P2

. . . . .P3

. . . . . .P4

.Wrap-up

Review of the second half

Rao-Blackwell : If W(X) is an unbiased estimator of τ(θ),ϕ(T) = E[W(X)|T] is a better unbiased estimator for asufficient statistic.

Uniqueness of MVUE : Theorem 7.3.19 - Best unbiased estimator isunique

MVUE and UE of zeros : Theorem 7.3.20 - Best unbiased estimator isuncorrelated with any unbiased estimators of zero

UMVE by complete sufficient statistics : Theorem 7.3.23 - Any functionof complete sufficient statistic is the best unbiased estimatorfor its expected value

How to get UMVUE Strategies to obtain best unbiased estimators:

• Condition a simple unbiased estimator on completesufficient statistics

• Come up with a function of sufficient statistic whoseexpected value is τ(θ).

Hyun Min Kang Biostatistics 602 - Lecture 26 Apil 23rd, 2013 2 / 31

Page 7: Biostatistics 602 - Statistical Inference Lecture 26 Final ... · Biostatistics 602 - Statistical Inference Lecture 26 Final Exam Review & Practice Problems for the Final Hyun Min

..........

.....

......

.....

.....

.....

......

.....

.....

.....

......

.....

.....

.....

......

.....

......

.....

.....

.

. . . . . . . .Review

. . . .P1

. . . . . .P2

. . . . .P3

. . . . . .P4

.Wrap-up

Review of the second half

Rao-Blackwell : If W(X) is an unbiased estimator of τ(θ),ϕ(T) = E[W(X)|T] is a better unbiased estimator for asufficient statistic.

Uniqueness of MVUE : Theorem 7.3.19 - Best unbiased estimator isunique

MVUE and UE of zeros : Theorem 7.3.20 - Best unbiased estimator isuncorrelated with any unbiased estimators of zero

UMVE by complete sufficient statistics : Theorem 7.3.23 - Any functionof complete sufficient statistic is the best unbiased estimatorfor its expected value

How to get UMVUE Strategies to obtain best unbiased estimators:• Condition a simple unbiased estimator on complete

sufficient statistics

• Come up with a function of sufficient statistic whoseexpected value is τ(θ).

Hyun Min Kang Biostatistics 602 - Lecture 26 Apil 23rd, 2013 2 / 31

Page 8: Biostatistics 602 - Statistical Inference Lecture 26 Final ... · Biostatistics 602 - Statistical Inference Lecture 26 Final Exam Review & Practice Problems for the Final Hyun Min

..........

.....

......

.....

.....

.....

......

.....

.....

.....

......

.....

.....

.....

......

.....

......

.....

.....

.

. . . . . . . .Review

. . . .P1

. . . . . .P2

. . . . .P3

. . . . . .P4

.Wrap-up

Review of the second half

Rao-Blackwell : If W(X) is an unbiased estimator of τ(θ),ϕ(T) = E[W(X)|T] is a better unbiased estimator for asufficient statistic.

Uniqueness of MVUE : Theorem 7.3.19 - Best unbiased estimator isunique

MVUE and UE of zeros : Theorem 7.3.20 - Best unbiased estimator isuncorrelated with any unbiased estimators of zero

UMVE by complete sufficient statistics : Theorem 7.3.23 - Any functionof complete sufficient statistic is the best unbiased estimatorfor its expected value

How to get UMVUE Strategies to obtain best unbiased estimators:• Condition a simple unbiased estimator on complete

sufficient statistics• Come up with a function of sufficient statistic whose

expected value is τ(θ).Hyun Min Kang Biostatistics 602 - Lecture 26 Apil 23rd, 2013 2 / 31

Page 9: Biostatistics 602 - Statistical Inference Lecture 26 Final ... · Biostatistics 602 - Statistical Inference Lecture 26 Final Exam Review & Practice Problems for the Final Hyun Min

..........

.....

......

.....

.....

.....

......

.....

.....

.....

......

.....

.....

.....

......

.....

......

.....

.....

.

. . . . . . . .Review

. . . .P1

. . . . . .P2

. . . . .P3

. . . . . .P4

.Wrap-up

Bayesian Framework

Prior distribution π(θ)

Sampling distribution x|θ ∼ fX(x|θ)Joint distribution π(θ)f(x|θ)Marginal distribution m(x) =

∫π(θ)f(x|θ)dθ

Posterior distribution π(θ|x) = fX(x|θ)π(θ)m(x)

Bayes Estimator is a posterior mean of θ : E[θ|x].

Hyun Min Kang Biostatistics 602 - Lecture 26 Apil 23rd, 2013 3 / 31

Page 10: Biostatistics 602 - Statistical Inference Lecture 26 Final ... · Biostatistics 602 - Statistical Inference Lecture 26 Final Exam Review & Practice Problems for the Final Hyun Min

..........

.....

......

.....

.....

.....

......

.....

.....

.....

......

.....

.....

.....

......

.....

......

.....

.....

.

. . . . . . . .Review

. . . .P1

. . . . . .P2

. . . . .P3

. . . . . .P4

.Wrap-up

Bayesian Framework

Prior distribution π(θ)

Sampling distribution x|θ ∼ fX(x|θ)

Joint distribution π(θ)f(x|θ)Marginal distribution m(x) =

∫π(θ)f(x|θ)dθ

Posterior distribution π(θ|x) = fX(x|θ)π(θ)m(x)

Bayes Estimator is a posterior mean of θ : E[θ|x].

Hyun Min Kang Biostatistics 602 - Lecture 26 Apil 23rd, 2013 3 / 31

Page 11: Biostatistics 602 - Statistical Inference Lecture 26 Final ... · Biostatistics 602 - Statistical Inference Lecture 26 Final Exam Review & Practice Problems for the Final Hyun Min

..........

.....

......

.....

.....

.....

......

.....

.....

.....

......

.....

.....

.....

......

.....

......

.....

.....

.

. . . . . . . .Review

. . . .P1

. . . . . .P2

. . . . .P3

. . . . . .P4

.Wrap-up

Bayesian Framework

Prior distribution π(θ)

Sampling distribution x|θ ∼ fX(x|θ)Joint distribution π(θ)f(x|θ)

Marginal distribution m(x) =∫π(θ)f(x|θ)dθ

Posterior distribution π(θ|x) = fX(x|θ)π(θ)m(x)

Bayes Estimator is a posterior mean of θ : E[θ|x].

Hyun Min Kang Biostatistics 602 - Lecture 26 Apil 23rd, 2013 3 / 31

Page 12: Biostatistics 602 - Statistical Inference Lecture 26 Final ... · Biostatistics 602 - Statistical Inference Lecture 26 Final Exam Review & Practice Problems for the Final Hyun Min

..........

.....

......

.....

.....

.....

......

.....

.....

.....

......

.....

.....

.....

......

.....

......

.....

.....

.

. . . . . . . .Review

. . . .P1

. . . . . .P2

. . . . .P3

. . . . . .P4

.Wrap-up

Bayesian Framework

Prior distribution π(θ)

Sampling distribution x|θ ∼ fX(x|θ)Joint distribution π(θ)f(x|θ)Marginal distribution m(x) =

∫π(θ)f(x|θ)dθ

Posterior distribution π(θ|x) = fX(x|θ)π(θ)m(x)

Bayes Estimator is a posterior mean of θ : E[θ|x].

Hyun Min Kang Biostatistics 602 - Lecture 26 Apil 23rd, 2013 3 / 31

Page 13: Biostatistics 602 - Statistical Inference Lecture 26 Final ... · Biostatistics 602 - Statistical Inference Lecture 26 Final Exam Review & Practice Problems for the Final Hyun Min

..........

.....

......

.....

.....

.....

......

.....

.....

.....

......

.....

.....

.....

......

.....

......

.....

.....

.

. . . . . . . .Review

. . . .P1

. . . . . .P2

. . . . .P3

. . . . . .P4

.Wrap-up

Bayesian Framework

Prior distribution π(θ)

Sampling distribution x|θ ∼ fX(x|θ)Joint distribution π(θ)f(x|θ)Marginal distribution m(x) =

∫π(θ)f(x|θ)dθ

Posterior distribution π(θ|x) = fX(x|θ)π(θ)m(x)

Bayes Estimator is a posterior mean of θ : E[θ|x].

Hyun Min Kang Biostatistics 602 - Lecture 26 Apil 23rd, 2013 3 / 31

Page 14: Biostatistics 602 - Statistical Inference Lecture 26 Final ... · Biostatistics 602 - Statistical Inference Lecture 26 Final Exam Review & Practice Problems for the Final Hyun Min

..........

.....

......

.....

.....

.....

......

.....

.....

.....

......

.....

.....

.....

......

.....

......

.....

.....

.

. . . . . . . .Review

. . . .P1

. . . . . .P2

. . . . .P3

. . . . . .P4

.Wrap-up

Bayesian Framework

Prior distribution π(θ)

Sampling distribution x|θ ∼ fX(x|θ)Joint distribution π(θ)f(x|θ)Marginal distribution m(x) =

∫π(θ)f(x|θ)dθ

Posterior distribution π(θ|x) = fX(x|θ)π(θ)m(x)

Bayes Estimator is a posterior mean of θ : E[θ|x].

Hyun Min Kang Biostatistics 602 - Lecture 26 Apil 23rd, 2013 3 / 31

Page 15: Biostatistics 602 - Statistical Inference Lecture 26 Final ... · Biostatistics 602 - Statistical Inference Lecture 26 Final Exam Review & Practice Problems for the Final Hyun Min

..........

.....

......

.....

.....

.....

......

.....

.....

.....

......

.....

.....

.....

......

.....

......

.....

.....

.

. . . . . . . .Review

. . . .P1

. . . . . .P2

. . . . .P3

. . . . . .P4

.Wrap-up

Bayesian Decision Theory

Loss Function L(θ, θ) (e.g. (θ − θ)2)

Risk Function is the average loss : R(θ, θ) = E[L(θ, θ)|θ].For squared error loss L = (θ − θ)2, the risk function is MSE

Bayes Risk is the average risk across all θ : E[R(θ, θ)|π(θ)].Bayes Rule Estimator minimizes Bayes risk ⇐⇒ minimizes posterior

expected loss.

Hyun Min Kang Biostatistics 602 - Lecture 26 Apil 23rd, 2013 4 / 31

Page 16: Biostatistics 602 - Statistical Inference Lecture 26 Final ... · Biostatistics 602 - Statistical Inference Lecture 26 Final Exam Review & Practice Problems for the Final Hyun Min

..........

.....

......

.....

.....

.....

......

.....

.....

.....

......

.....

.....

.....

......

.....

......

.....

.....

.

. . . . . . . .Review

. . . .P1

. . . . . .P2

. . . . .P3

. . . . . .P4

.Wrap-up

Bayesian Decision Theory

Loss Function L(θ, θ) (e.g. (θ − θ)2)Risk Function is the average loss : R(θ, θ) = E[L(θ, θ)|θ].

For squared error loss L = (θ − θ)2, the risk function is MSEBayes Risk is the average risk across all θ : E[R(θ, θ)|π(θ)].

Bayes Rule Estimator minimizes Bayes risk ⇐⇒ minimizes posteriorexpected loss.

Hyun Min Kang Biostatistics 602 - Lecture 26 Apil 23rd, 2013 4 / 31

Page 17: Biostatistics 602 - Statistical Inference Lecture 26 Final ... · Biostatistics 602 - Statistical Inference Lecture 26 Final Exam Review & Practice Problems for the Final Hyun Min

..........

.....

......

.....

.....

.....

......

.....

.....

.....

......

.....

.....

.....

......

.....

......

.....

.....

.

. . . . . . . .Review

. . . .P1

. . . . . .P2

. . . . .P3

. . . . . .P4

.Wrap-up

Bayesian Decision Theory

Loss Function L(θ, θ) (e.g. (θ − θ)2)Risk Function is the average loss : R(θ, θ) = E[L(θ, θ)|θ].

For squared error loss L = (θ − θ)2, the risk function is MSE

Bayes Risk is the average risk across all θ : E[R(θ, θ)|π(θ)].Bayes Rule Estimator minimizes Bayes risk ⇐⇒ minimizes posterior

expected loss.

Hyun Min Kang Biostatistics 602 - Lecture 26 Apil 23rd, 2013 4 / 31

Page 18: Biostatistics 602 - Statistical Inference Lecture 26 Final ... · Biostatistics 602 - Statistical Inference Lecture 26 Final Exam Review & Practice Problems for the Final Hyun Min

..........

.....

......

.....

.....

.....

......

.....

.....

.....

......

.....

.....

.....

......

.....

......

.....

.....

.

. . . . . . . .Review

. . . .P1

. . . . . .P2

. . . . .P3

. . . . . .P4

.Wrap-up

Bayesian Decision Theory

Loss Function L(θ, θ) (e.g. (θ − θ)2)Risk Function is the average loss : R(θ, θ) = E[L(θ, θ)|θ].

For squared error loss L = (θ − θ)2, the risk function is MSEBayes Risk is the average risk across all θ : E[R(θ, θ)|π(θ)].

Bayes Rule Estimator minimizes Bayes risk ⇐⇒ minimizes posteriorexpected loss.

Hyun Min Kang Biostatistics 602 - Lecture 26 Apil 23rd, 2013 4 / 31

Page 19: Biostatistics 602 - Statistical Inference Lecture 26 Final ... · Biostatistics 602 - Statistical Inference Lecture 26 Final Exam Review & Practice Problems for the Final Hyun Min

..........

.....

......

.....

.....

.....

......

.....

.....

.....

......

.....

.....

.....

......

.....

......

.....

.....

.

. . . . . . . .Review

. . . .P1

. . . . . .P2

. . . . .P3

. . . . . .P4

.Wrap-up

Bayesian Decision Theory

Loss Function L(θ, θ) (e.g. (θ − θ)2)Risk Function is the average loss : R(θ, θ) = E[L(θ, θ)|θ].

For squared error loss L = (θ − θ)2, the risk function is MSEBayes Risk is the average risk across all θ : E[R(θ, θ)|π(θ)].

Bayes Rule Estimator minimizes Bayes risk ⇐⇒ minimizes posteriorexpected loss.

Hyun Min Kang Biostatistics 602 - Lecture 26 Apil 23rd, 2013 4 / 31

Page 20: Biostatistics 602 - Statistical Inference Lecture 26 Final ... · Biostatistics 602 - Statistical Inference Lecture 26 Final Exam Review & Practice Problems for the Final Hyun Min

..........

.....

......

.....

.....

.....

......

.....

.....

.....

......

.....

.....

.....

......

.....

......

.....

.....

.

. . . . . . . .Review

. . . .P1

. . . . . .P2

. . . . .P3

. . . . . .P4

.Wrap-up

Asymptotics

Consistency Using law of large numbers, show variance and biasconverges to zero, for any continuous mapping function τ

Asymptotic Normality Using central limit theorem, Slutsky Theorem, andDelta Method

Asymptotic Relative Efficiency ARE(Vn,Wn) = σ2W/σ2

V.Asymptotically Efficient ARE with CR-bound of unbiased estimator of

τ(θ) is 1.Asymptotic Efficiency of MLE Theorem 10.1.12 MLE is always

asymptotically efficient under regularity condition.

Hyun Min Kang Biostatistics 602 - Lecture 26 Apil 23rd, 2013 5 / 31

Page 21: Biostatistics 602 - Statistical Inference Lecture 26 Final ... · Biostatistics 602 - Statistical Inference Lecture 26 Final Exam Review & Practice Problems for the Final Hyun Min

..........

.....

......

.....

.....

.....

......

.....

.....

.....

......

.....

.....

.....

......

.....

......

.....

.....

.

. . . . . . . .Review

. . . .P1

. . . . . .P2

. . . . .P3

. . . . . .P4

.Wrap-up

Asymptotics

Consistency Using law of large numbers, show variance and biasconverges to zero, for any continuous mapping function τ

Asymptotic Normality Using central limit theorem, Slutsky Theorem, andDelta Method

Asymptotic Relative Efficiency ARE(Vn,Wn) = σ2W/σ2

V.Asymptotically Efficient ARE with CR-bound of unbiased estimator of

τ(θ) is 1.Asymptotic Efficiency of MLE Theorem 10.1.12 MLE is always

asymptotically efficient under regularity condition.

Hyun Min Kang Biostatistics 602 - Lecture 26 Apil 23rd, 2013 5 / 31

Page 22: Biostatistics 602 - Statistical Inference Lecture 26 Final ... · Biostatistics 602 - Statistical Inference Lecture 26 Final Exam Review & Practice Problems for the Final Hyun Min

..........

.....

......

.....

.....

.....

......

.....

.....

.....

......

.....

.....

.....

......

.....

......

.....

.....

.

. . . . . . . .Review

. . . .P1

. . . . . .P2

. . . . .P3

. . . . . .P4

.Wrap-up

Asymptotics

Consistency Using law of large numbers, show variance and biasconverges to zero, for any continuous mapping function τ

Asymptotic Normality Using central limit theorem, Slutsky Theorem, andDelta Method

Asymptotic Relative Efficiency ARE(Vn,Wn) = σ2W/σ2

V.

Asymptotically Efficient ARE with CR-bound of unbiased estimator ofτ(θ) is 1.

Asymptotic Efficiency of MLE Theorem 10.1.12 MLE is alwaysasymptotically efficient under regularity condition.

Hyun Min Kang Biostatistics 602 - Lecture 26 Apil 23rd, 2013 5 / 31

Page 23: Biostatistics 602 - Statistical Inference Lecture 26 Final ... · Biostatistics 602 - Statistical Inference Lecture 26 Final Exam Review & Practice Problems for the Final Hyun Min

..........

.....

......

.....

.....

.....

......

.....

.....

.....

......

.....

.....

.....

......

.....

......

.....

.....

.

. . . . . . . .Review

. . . .P1

. . . . . .P2

. . . . .P3

. . . . . .P4

.Wrap-up

Asymptotics

Consistency Using law of large numbers, show variance and biasconverges to zero, for any continuous mapping function τ

Asymptotic Normality Using central limit theorem, Slutsky Theorem, andDelta Method

Asymptotic Relative Efficiency ARE(Vn,Wn) = σ2W/σ2

V.Asymptotically Efficient ARE with CR-bound of unbiased estimator of

τ(θ) is 1.

Asymptotic Efficiency of MLE Theorem 10.1.12 MLE is alwaysasymptotically efficient under regularity condition.

Hyun Min Kang Biostatistics 602 - Lecture 26 Apil 23rd, 2013 5 / 31

Page 24: Biostatistics 602 - Statistical Inference Lecture 26 Final ... · Biostatistics 602 - Statistical Inference Lecture 26 Final Exam Review & Practice Problems for the Final Hyun Min

..........

.....

......

.....

.....

.....

......

.....

.....

.....

......

.....

.....

.....

......

.....

......

.....

.....

.

. . . . . . . .Review

. . . .P1

. . . . . .P2

. . . . .P3

. . . . . .P4

.Wrap-up

Asymptotics

Consistency Using law of large numbers, show variance and biasconverges to zero, for any continuous mapping function τ

Asymptotic Normality Using central limit theorem, Slutsky Theorem, andDelta Method

Asymptotic Relative Efficiency ARE(Vn,Wn) = σ2W/σ2

V.Asymptotically Efficient ARE with CR-bound of unbiased estimator of

τ(θ) is 1.Asymptotic Efficiency of MLE Theorem 10.1.12 MLE is always

asymptotically efficient under regularity condition.

Hyun Min Kang Biostatistics 602 - Lecture 26 Apil 23rd, 2013 5 / 31

Page 25: Biostatistics 602 - Statistical Inference Lecture 26 Final ... · Biostatistics 602 - Statistical Inference Lecture 26 Final Exam Review & Practice Problems for the Final Hyun Min

..........

.....

......

.....

.....

.....

......

.....

.....

.....

......

.....

.....

.....

......

.....

......

.....

.....

.

. . . . . . . .Review

. . . .P1

. . . . . .P2

. . . . .P3

. . . . . .P4

.Wrap-up

Hypothesis Testing

Type I error Pr(X ∈ R|θ) when θ ∈ Ω0

Type II error 1− Pr(X ∈ R|θ) when θ ∈ Ωc0

Power function β(θ) = Pr(X ∈ R|θ)β(θ) represents Type I error under H0, and power (=1-TypeII error) under H1.

Size α test supθ∈Ω0β(θ) = α

Level α test supθ∈Ω0β(θ) ≤ α

LRT λ(x) = L(θ0|x)L(θ|x)

rejects H0 when λ(x) ≤ c

⇐⇒ −2 logλ(x) ≥ −2 log c = c∗

LRT based on sufficient statistics LRT based on full data and sufficientstatistics are identical.

Hyun Min Kang Biostatistics 602 - Lecture 26 Apil 23rd, 2013 6 / 31

Page 26: Biostatistics 602 - Statistical Inference Lecture 26 Final ... · Biostatistics 602 - Statistical Inference Lecture 26 Final Exam Review & Practice Problems for the Final Hyun Min

..........

.....

......

.....

.....

.....

......

.....

.....

.....

......

.....

.....

.....

......

.....

......

.....

.....

.

. . . . . . . .Review

. . . .P1

. . . . . .P2

. . . . .P3

. . . . . .P4

.Wrap-up

Hypothesis Testing

Type I error Pr(X ∈ R|θ) when θ ∈ Ω0

Type II error 1− Pr(X ∈ R|θ) when θ ∈ Ωc0

Power function β(θ) = Pr(X ∈ R|θ)β(θ) represents Type I error under H0, and power (=1-TypeII error) under H1.

Size α test supθ∈Ω0β(θ) = α

Level α test supθ∈Ω0β(θ) ≤ α

LRT λ(x) = L(θ0|x)L(θ|x)

rejects H0 when λ(x) ≤ c

⇐⇒ −2 logλ(x) ≥ −2 log c = c∗

LRT based on sufficient statistics LRT based on full data and sufficientstatistics are identical.

Hyun Min Kang Biostatistics 602 - Lecture 26 Apil 23rd, 2013 6 / 31

Page 27: Biostatistics 602 - Statistical Inference Lecture 26 Final ... · Biostatistics 602 - Statistical Inference Lecture 26 Final Exam Review & Practice Problems for the Final Hyun Min

..........

.....

......

.....

.....

.....

......

.....

.....

.....

......

.....

.....

.....

......

.....

......

.....

.....

.

. . . . . . . .Review

. . . .P1

. . . . . .P2

. . . . .P3

. . . . . .P4

.Wrap-up

Hypothesis Testing

Type I error Pr(X ∈ R|θ) when θ ∈ Ω0

Type II error 1− Pr(X ∈ R|θ) when θ ∈ Ωc0

Power function β(θ) = Pr(X ∈ R|θ)

β(θ) represents Type I error under H0, and power (=1-TypeII error) under H1.

Size α test supθ∈Ω0β(θ) = α

Level α test supθ∈Ω0β(θ) ≤ α

LRT λ(x) = L(θ0|x)L(θ|x)

rejects H0 when λ(x) ≤ c

⇐⇒ −2 logλ(x) ≥ −2 log c = c∗

LRT based on sufficient statistics LRT based on full data and sufficientstatistics are identical.

Hyun Min Kang Biostatistics 602 - Lecture 26 Apil 23rd, 2013 6 / 31

Page 28: Biostatistics 602 - Statistical Inference Lecture 26 Final ... · Biostatistics 602 - Statistical Inference Lecture 26 Final Exam Review & Practice Problems for the Final Hyun Min

..........

.....

......

.....

.....

.....

......

.....

.....

.....

......

.....

.....

.....

......

.....

......

.....

.....

.

. . . . . . . .Review

. . . .P1

. . . . . .P2

. . . . .P3

. . . . . .P4

.Wrap-up

Hypothesis Testing

Type I error Pr(X ∈ R|θ) when θ ∈ Ω0

Type II error 1− Pr(X ∈ R|θ) when θ ∈ Ωc0

Power function β(θ) = Pr(X ∈ R|θ)β(θ) represents Type I error under H0, and power (=1-TypeII error) under H1.

Size α test supθ∈Ω0β(θ) = α

Level α test supθ∈Ω0β(θ) ≤ α

LRT λ(x) = L(θ0|x)L(θ|x)

rejects H0 when λ(x) ≤ c

⇐⇒ −2 logλ(x) ≥ −2 log c = c∗

LRT based on sufficient statistics LRT based on full data and sufficientstatistics are identical.

Hyun Min Kang Biostatistics 602 - Lecture 26 Apil 23rd, 2013 6 / 31

Page 29: Biostatistics 602 - Statistical Inference Lecture 26 Final ... · Biostatistics 602 - Statistical Inference Lecture 26 Final Exam Review & Practice Problems for the Final Hyun Min

..........

.....

......

.....

.....

.....

......

.....

.....

.....

......

.....

.....

.....

......

.....

......

.....

.....

.

. . . . . . . .Review

. . . .P1

. . . . . .P2

. . . . .P3

. . . . . .P4

.Wrap-up

Hypothesis Testing

Type I error Pr(X ∈ R|θ) when θ ∈ Ω0

Type II error 1− Pr(X ∈ R|θ) when θ ∈ Ωc0

Power function β(θ) = Pr(X ∈ R|θ)β(θ) represents Type I error under H0, and power (=1-TypeII error) under H1.

Size α test supθ∈Ω0β(θ) = α

Level α test supθ∈Ω0β(θ) ≤ α

LRT λ(x) = L(θ0|x)L(θ|x)

rejects H0 when λ(x) ≤ c

⇐⇒ −2 logλ(x) ≥ −2 log c = c∗

LRT based on sufficient statistics LRT based on full data and sufficientstatistics are identical.

Hyun Min Kang Biostatistics 602 - Lecture 26 Apil 23rd, 2013 6 / 31

Page 30: Biostatistics 602 - Statistical Inference Lecture 26 Final ... · Biostatistics 602 - Statistical Inference Lecture 26 Final Exam Review & Practice Problems for the Final Hyun Min

..........

.....

......

.....

.....

.....

......

.....

.....

.....

......

.....

.....

.....

......

.....

......

.....

.....

.

. . . . . . . .Review

. . . .P1

. . . . . .P2

. . . . .P3

. . . . . .P4

.Wrap-up

Hypothesis Testing

Type I error Pr(X ∈ R|θ) when θ ∈ Ω0

Type II error 1− Pr(X ∈ R|θ) when θ ∈ Ωc0

Power function β(θ) = Pr(X ∈ R|θ)β(θ) represents Type I error under H0, and power (=1-TypeII error) under H1.

Size α test supθ∈Ω0β(θ) = α

Level α test supθ∈Ω0β(θ) ≤ α

LRT λ(x) = L(θ0|x)L(θ|x)

rejects H0 when λ(x) ≤ c

⇐⇒ −2 logλ(x) ≥ −2 log c = c∗

LRT based on sufficient statistics LRT based on full data and sufficientstatistics are identical.

Hyun Min Kang Biostatistics 602 - Lecture 26 Apil 23rd, 2013 6 / 31

Page 31: Biostatistics 602 - Statistical Inference Lecture 26 Final ... · Biostatistics 602 - Statistical Inference Lecture 26 Final Exam Review & Practice Problems for the Final Hyun Min

..........

.....

......

.....

.....

.....

......

.....

.....

.....

......

.....

.....

.....

......

.....

......

.....

.....

.

. . . . . . . .Review

. . . .P1

. . . . . .P2

. . . . .P3

. . . . . .P4

.Wrap-up

Hypothesis Testing

Type I error Pr(X ∈ R|θ) when θ ∈ Ω0

Type II error 1− Pr(X ∈ R|θ) when θ ∈ Ωc0

Power function β(θ) = Pr(X ∈ R|θ)β(θ) represents Type I error under H0, and power (=1-TypeII error) under H1.

Size α test supθ∈Ω0β(θ) = α

Level α test supθ∈Ω0β(θ) ≤ α

LRT λ(x) = L(θ0|x)L(θ|x)

rejects H0 when λ(x) ≤ c

⇐⇒ −2 logλ(x) ≥ −2 log c = c∗

LRT based on sufficient statistics LRT based on full data and sufficientstatistics are identical.

Hyun Min Kang Biostatistics 602 - Lecture 26 Apil 23rd, 2013 6 / 31

Page 32: Biostatistics 602 - Statistical Inference Lecture 26 Final ... · Biostatistics 602 - Statistical Inference Lecture 26 Final Exam Review & Practice Problems for the Final Hyun Min

..........

.....

......

.....

.....

.....

......

.....

.....

.....

......

.....

.....

.....

......

.....

......

.....

.....

.

. . . . . . . .Review

. . . .P1

. . . . . .P2

. . . . .P3

. . . . . .P4

.Wrap-up

Hypothesis Testing

Type I error Pr(X ∈ R|θ) when θ ∈ Ω0

Type II error 1− Pr(X ∈ R|θ) when θ ∈ Ωc0

Power function β(θ) = Pr(X ∈ R|θ)β(θ) represents Type I error under H0, and power (=1-TypeII error) under H1.

Size α test supθ∈Ω0β(θ) = α

Level α test supθ∈Ω0β(θ) ≤ α

LRT λ(x) = L(θ0|x)L(θ|x)

rejects H0 when λ(x) ≤ c

⇐⇒ −2 logλ(x) ≥ −2 log c = c∗

LRT based on sufficient statistics LRT based on full data and sufficientstatistics are identical.

Hyun Min Kang Biostatistics 602 - Lecture 26 Apil 23rd, 2013 6 / 31

Page 33: Biostatistics 602 - Statistical Inference Lecture 26 Final ... · Biostatistics 602 - Statistical Inference Lecture 26 Final Exam Review & Practice Problems for the Final Hyun Min

..........

.....

......

.....

.....

.....

......

.....

.....

.....

......

.....

.....

.....

......

.....

......

.....

.....

.

. . . . . . . .Review

. . . .P1

. . . . . .P2

. . . . .P3

. . . . . .P4

.Wrap-up

Hypothesis Testing

Type I error Pr(X ∈ R|θ) when θ ∈ Ω0

Type II error 1− Pr(X ∈ R|θ) when θ ∈ Ωc0

Power function β(θ) = Pr(X ∈ R|θ)β(θ) represents Type I error under H0, and power (=1-TypeII error) under H1.

Size α test supθ∈Ω0β(θ) = α

Level α test supθ∈Ω0β(θ) ≤ α

LRT λ(x) = L(θ0|x)L(θ|x)

rejects H0 when λ(x) ≤ c

⇐⇒ −2 logλ(x) ≥ −2 log c = c∗

LRT based on sufficient statistics LRT based on full data and sufficientstatistics are identical.

Hyun Min Kang Biostatistics 602 - Lecture 26 Apil 23rd, 2013 6 / 31

Page 34: Biostatistics 602 - Statistical Inference Lecture 26 Final ... · Biostatistics 602 - Statistical Inference Lecture 26 Final Exam Review & Practice Problems for the Final Hyun Min

..........

.....

......

.....

.....

.....

......

.....

.....

.....

......

.....

.....

.....

......

.....

......

.....

.....

.

. . . . . . . .Review

. . . .P1

. . . . . .P2

. . . . .P3

. . . . . .P4

.Wrap-up

UMP

Unbiased Test β(θ1) ≥ β(θ0) for every θ1 ∈ Ωc0 and θ0 ∈ Ω0.

UMP Test β(θ) ≥ β′(θ) for every θ ∈ Ωc0 and β′(θ) of every other test

with a class of test C.UMP level α Test UMP test in the class of all the level α test. (smallest

Type II error given the upper bound of Type I error)Neyman-Pearson For H0 : θ = θ0 vs. H1 : θ = θ1, a test with rejection

region f(x|θ1)/f(x|θ0) > k is a UMP level α test for its size.MLR g(t|θ2)/g(t|θ1) is an increasing function of t for every

θ2 > θ1.Karlin-Rabin If T is sufficient and has MLR, then test rejecting

R = T : T > t0 or R = T : T < t0 is an UMP level αtest for one-sided composite hypothesis.

Hyun Min Kang Biostatistics 602 - Lecture 26 Apil 23rd, 2013 7 / 31

Page 35: Biostatistics 602 - Statistical Inference Lecture 26 Final ... · Biostatistics 602 - Statistical Inference Lecture 26 Final Exam Review & Practice Problems for the Final Hyun Min

..........

.....

......

.....

.....

.....

......

.....

.....

.....

......

.....

.....

.....

......

.....

......

.....

.....

.

. . . . . . . .Review

. . . .P1

. . . . . .P2

. . . . .P3

. . . . . .P4

.Wrap-up

UMP

Unbiased Test β(θ1) ≥ β(θ0) for every θ1 ∈ Ωc0 and θ0 ∈ Ω0.

UMP Test β(θ) ≥ β′(θ) for every θ ∈ Ωc0 and β′(θ) of every other test

with a class of test C.

UMP level α Test UMP test in the class of all the level α test. (smallestType II error given the upper bound of Type I error)

Neyman-Pearson For H0 : θ = θ0 vs. H1 : θ = θ1, a test with rejectionregion f(x|θ1)/f(x|θ0) > k is a UMP level α test for its size.

MLR g(t|θ2)/g(t|θ1) is an increasing function of t for everyθ2 > θ1.

Karlin-Rabin If T is sufficient and has MLR, then test rejectingR = T : T > t0 or R = T : T < t0 is an UMP level αtest for one-sided composite hypothesis.

Hyun Min Kang Biostatistics 602 - Lecture 26 Apil 23rd, 2013 7 / 31

Page 36: Biostatistics 602 - Statistical Inference Lecture 26 Final ... · Biostatistics 602 - Statistical Inference Lecture 26 Final Exam Review & Practice Problems for the Final Hyun Min

..........

.....

......

.....

.....

.....

......

.....

.....

.....

......

.....

.....

.....

......

.....

......

.....

.....

.

. . . . . . . .Review

. . . .P1

. . . . . .P2

. . . . .P3

. . . . . .P4

.Wrap-up

UMP

Unbiased Test β(θ1) ≥ β(θ0) for every θ1 ∈ Ωc0 and θ0 ∈ Ω0.

UMP Test β(θ) ≥ β′(θ) for every θ ∈ Ωc0 and β′(θ) of every other test

with a class of test C.UMP level α Test UMP test in the class of all the level α test. (smallest

Type II error given the upper bound of Type I error)

Neyman-Pearson For H0 : θ = θ0 vs. H1 : θ = θ1, a test with rejectionregion f(x|θ1)/f(x|θ0) > k is a UMP level α test for its size.

MLR g(t|θ2)/g(t|θ1) is an increasing function of t for everyθ2 > θ1.

Karlin-Rabin If T is sufficient and has MLR, then test rejectingR = T : T > t0 or R = T : T < t0 is an UMP level αtest for one-sided composite hypothesis.

Hyun Min Kang Biostatistics 602 - Lecture 26 Apil 23rd, 2013 7 / 31

Page 37: Biostatistics 602 - Statistical Inference Lecture 26 Final ... · Biostatistics 602 - Statistical Inference Lecture 26 Final Exam Review & Practice Problems for the Final Hyun Min

..........

.....

......

.....

.....

.....

......

.....

.....

.....

......

.....

.....

.....

......

.....

......

.....

.....

.

. . . . . . . .Review

. . . .P1

. . . . . .P2

. . . . .P3

. . . . . .P4

.Wrap-up

UMP

Unbiased Test β(θ1) ≥ β(θ0) for every θ1 ∈ Ωc0 and θ0 ∈ Ω0.

UMP Test β(θ) ≥ β′(θ) for every θ ∈ Ωc0 and β′(θ) of every other test

with a class of test C.UMP level α Test UMP test in the class of all the level α test. (smallest

Type II error given the upper bound of Type I error)Neyman-Pearson For H0 : θ = θ0 vs. H1 : θ = θ1, a test with rejection

region f(x|θ1)/f(x|θ0) > k is a UMP level α test for its size.

MLR g(t|θ2)/g(t|θ1) is an increasing function of t for everyθ2 > θ1.

Karlin-Rabin If T is sufficient and has MLR, then test rejectingR = T : T > t0 or R = T : T < t0 is an UMP level αtest for one-sided composite hypothesis.

Hyun Min Kang Biostatistics 602 - Lecture 26 Apil 23rd, 2013 7 / 31

Page 38: Biostatistics 602 - Statistical Inference Lecture 26 Final ... · Biostatistics 602 - Statistical Inference Lecture 26 Final Exam Review & Practice Problems for the Final Hyun Min

..........

.....

......

.....

.....

.....

......

.....

.....

.....

......

.....

.....

.....

......

.....

......

.....

.....

.

. . . . . . . .Review

. . . .P1

. . . . . .P2

. . . . .P3

. . . . . .P4

.Wrap-up

UMP

Unbiased Test β(θ1) ≥ β(θ0) for every θ1 ∈ Ωc0 and θ0 ∈ Ω0.

UMP Test β(θ) ≥ β′(θ) for every θ ∈ Ωc0 and β′(θ) of every other test

with a class of test C.UMP level α Test UMP test in the class of all the level α test. (smallest

Type II error given the upper bound of Type I error)Neyman-Pearson For H0 : θ = θ0 vs. H1 : θ = θ1, a test with rejection

region f(x|θ1)/f(x|θ0) > k is a UMP level α test for its size.MLR g(t|θ2)/g(t|θ1) is an increasing function of t for every

θ2 > θ1.

Karlin-Rabin If T is sufficient and has MLR, then test rejectingR = T : T > t0 or R = T : T < t0 is an UMP level αtest for one-sided composite hypothesis.

Hyun Min Kang Biostatistics 602 - Lecture 26 Apil 23rd, 2013 7 / 31

Page 39: Biostatistics 602 - Statistical Inference Lecture 26 Final ... · Biostatistics 602 - Statistical Inference Lecture 26 Final Exam Review & Practice Problems for the Final Hyun Min

..........

.....

......

.....

.....

.....

......

.....

.....

.....

......

.....

.....

.....

......

.....

......

.....

.....

.

. . . . . . . .Review

. . . .P1

. . . . . .P2

. . . . .P3

. . . . . .P4

.Wrap-up

UMP

Unbiased Test β(θ1) ≥ β(θ0) for every θ1 ∈ Ωc0 and θ0 ∈ Ω0.

UMP Test β(θ) ≥ β′(θ) for every θ ∈ Ωc0 and β′(θ) of every other test

with a class of test C.UMP level α Test UMP test in the class of all the level α test. (smallest

Type II error given the upper bound of Type I error)Neyman-Pearson For H0 : θ = θ0 vs. H1 : θ = θ1, a test with rejection

region f(x|θ1)/f(x|θ0) > k is a UMP level α test for its size.MLR g(t|θ2)/g(t|θ1) is an increasing function of t for every

θ2 > θ1.Karlin-Rabin If T is sufficient and has MLR, then test rejecting

R = T : T > t0 or R = T : T < t0 is an UMP level αtest for one-sided composite hypothesis.

Hyun Min Kang Biostatistics 602 - Lecture 26 Apil 23rd, 2013 7 / 31

Page 40: Biostatistics 602 - Statistical Inference Lecture 26 Final ... · Biostatistics 602 - Statistical Inference Lecture 26 Final Exam Review & Practice Problems for the Final Hyun Min

..........

.....

......

.....

.....

.....

......

.....

.....

.....

......

.....

.....

.....

......

.....

......

.....

.....

.

. . . . . . . .Review

. . . .P1

. . . . . .P2

. . . . .P3

. . . . . .P4

.Wrap-up

Asymptotic Tests and p-Values

Asymptotic Distribution of LRT For testing, H0 : θ = θ0 vs. H1 : θ = θ1,−2 logλ(x) d→ χ2

1 under regularity condition.

Wald Test If Wn is a consistent estimator of θ, and S2n is a consistent

estimator of Var(Wn), then Zn = (Wn − θ0)/Sn follows astandard normal distribution

• Two-sided test : |Zn| > zα/2• One-sided test : Zn > zα/2 or Zn < −zα/2

p-Value A p-value 0 ≤ p(x) ≤ 1 is valid if, Pr(p(X) ≤ α|θ) ≤ α forevery θ ∈ Ω0 and 0 ≤ α ≤ 1.

Constructing p-Value Theorem 8.3.27 : If large W(X) value gives evidencethat H1 is true, p(x) = supθ∈Ω0

Pr(W(X) ≥ W(x)|θ) is avalid p-value

p-Value given sufficient statistics For a sufficient statistic S(X),p(x) = Pr(W(X) ≥ W(x)|S(X) = S(x)) is also a validp-value.

Hyun Min Kang Biostatistics 602 - Lecture 26 Apil 23rd, 2013 8 / 31

Page 41: Biostatistics 602 - Statistical Inference Lecture 26 Final ... · Biostatistics 602 - Statistical Inference Lecture 26 Final Exam Review & Practice Problems for the Final Hyun Min

..........

.....

......

.....

.....

.....

......

.....

.....

.....

......

.....

.....

.....

......

.....

......

.....

.....

.

. . . . . . . .Review

. . . .P1

. . . . . .P2

. . . . .P3

. . . . . .P4

.Wrap-up

Asymptotic Tests and p-Values

Asymptotic Distribution of LRT For testing, H0 : θ = θ0 vs. H1 : θ = θ1,−2 logλ(x) d→ χ2

1 under regularity condition.Wald Test If Wn is a consistent estimator of θ, and S2

n is a consistentestimator of Var(Wn), then Zn = (Wn − θ0)/Sn follows astandard normal distribution

• Two-sided test : |Zn| > zα/2• One-sided test : Zn > zα/2 or Zn < −zα/2

p-Value A p-value 0 ≤ p(x) ≤ 1 is valid if, Pr(p(X) ≤ α|θ) ≤ α forevery θ ∈ Ω0 and 0 ≤ α ≤ 1.

Constructing p-Value Theorem 8.3.27 : If large W(X) value gives evidencethat H1 is true, p(x) = supθ∈Ω0

Pr(W(X) ≥ W(x)|θ) is avalid p-value

p-Value given sufficient statistics For a sufficient statistic S(X),p(x) = Pr(W(X) ≥ W(x)|S(X) = S(x)) is also a validp-value.

Hyun Min Kang Biostatistics 602 - Lecture 26 Apil 23rd, 2013 8 / 31

Page 42: Biostatistics 602 - Statistical Inference Lecture 26 Final ... · Biostatistics 602 - Statistical Inference Lecture 26 Final Exam Review & Practice Problems for the Final Hyun Min

..........

.....

......

.....

.....

.....

......

.....

.....

.....

......

.....

.....

.....

......

.....

......

.....

.....

.

. . . . . . . .Review

. . . .P1

. . . . . .P2

. . . . .P3

. . . . . .P4

.Wrap-up

Asymptotic Tests and p-Values

Asymptotic Distribution of LRT For testing, H0 : θ = θ0 vs. H1 : θ = θ1,−2 logλ(x) d→ χ2

1 under regularity condition.Wald Test If Wn is a consistent estimator of θ, and S2

n is a consistentestimator of Var(Wn), then Zn = (Wn − θ0)/Sn follows astandard normal distribution

• Two-sided test : |Zn| > zα/2• One-sided test : Zn > zα/2 or Zn < −zα/2

p-Value A p-value 0 ≤ p(x) ≤ 1 is valid if, Pr(p(X) ≤ α|θ) ≤ α forevery θ ∈ Ω0 and 0 ≤ α ≤ 1.

Constructing p-Value Theorem 8.3.27 : If large W(X) value gives evidencethat H1 is true, p(x) = supθ∈Ω0

Pr(W(X) ≥ W(x)|θ) is avalid p-value

p-Value given sufficient statistics For a sufficient statistic S(X),p(x) = Pr(W(X) ≥ W(x)|S(X) = S(x)) is also a validp-value.

Hyun Min Kang Biostatistics 602 - Lecture 26 Apil 23rd, 2013 8 / 31

Page 43: Biostatistics 602 - Statistical Inference Lecture 26 Final ... · Biostatistics 602 - Statistical Inference Lecture 26 Final Exam Review & Practice Problems for the Final Hyun Min

..........

.....

......

.....

.....

.....

......

.....

.....

.....

......

.....

.....

.....

......

.....

......

.....

.....

.

. . . . . . . .Review

. . . .P1

. . . . . .P2

. . . . .P3

. . . . . .P4

.Wrap-up

Asymptotic Tests and p-Values

Asymptotic Distribution of LRT For testing, H0 : θ = θ0 vs. H1 : θ = θ1,−2 logλ(x) d→ χ2

1 under regularity condition.Wald Test If Wn is a consistent estimator of θ, and S2

n is a consistentestimator of Var(Wn), then Zn = (Wn − θ0)/Sn follows astandard normal distribution

• Two-sided test : |Zn| > zα/2• One-sided test : Zn > zα/2 or Zn < −zα/2

p-Value A p-value 0 ≤ p(x) ≤ 1 is valid if, Pr(p(X) ≤ α|θ) ≤ α forevery θ ∈ Ω0 and 0 ≤ α ≤ 1.

Constructing p-Value Theorem 8.3.27 : If large W(X) value gives evidencethat H1 is true, p(x) = supθ∈Ω0

Pr(W(X) ≥ W(x)|θ) is avalid p-value

p-Value given sufficient statistics For a sufficient statistic S(X),p(x) = Pr(W(X) ≥ W(x)|S(X) = S(x)) is also a validp-value.

Hyun Min Kang Biostatistics 602 - Lecture 26 Apil 23rd, 2013 8 / 31

Page 44: Biostatistics 602 - Statistical Inference Lecture 26 Final ... · Biostatistics 602 - Statistical Inference Lecture 26 Final Exam Review & Practice Problems for the Final Hyun Min

..........

.....

......

.....

.....

.....

......

.....

.....

.....

......

.....

.....

.....

......

.....

......

.....

.....

.

. . . . . . . .Review

. . . .P1

. . . . . .P2

. . . . .P3

. . . . . .P4

.Wrap-up

Asymptotic Tests and p-Values

Asymptotic Distribution of LRT For testing, H0 : θ = θ0 vs. H1 : θ = θ1,−2 logλ(x) d→ χ2

1 under regularity condition.Wald Test If Wn is a consistent estimator of θ, and S2

n is a consistentestimator of Var(Wn), then Zn = (Wn − θ0)/Sn follows astandard normal distribution

• Two-sided test : |Zn| > zα/2• One-sided test : Zn > zα/2 or Zn < −zα/2

p-Value A p-value 0 ≤ p(x) ≤ 1 is valid if, Pr(p(X) ≤ α|θ) ≤ α forevery θ ∈ Ω0 and 0 ≤ α ≤ 1.

Constructing p-Value Theorem 8.3.27 : If large W(X) value gives evidencethat H1 is true, p(x) = supθ∈Ω0

Pr(W(X) ≥ W(x)|θ) is avalid p-value

p-Value given sufficient statistics For a sufficient statistic S(X),p(x) = Pr(W(X) ≥ W(x)|S(X) = S(x)) is also a validp-value.

Hyun Min Kang Biostatistics 602 - Lecture 26 Apil 23rd, 2013 8 / 31

Page 45: Biostatistics 602 - Statistical Inference Lecture 26 Final ... · Biostatistics 602 - Statistical Inference Lecture 26 Final Exam Review & Practice Problems for the Final Hyun Min

..........

.....

......

.....

.....

.....

......

.....

.....

.....

......

.....

.....

.....

......

.....

......

.....

.....

.

. . . . . . . .Review

. . . .P1

. . . . . .P2

. . . . .P3

. . . . . .P4

.Wrap-up

Asymptotic Tests and p-Values

Asymptotic Distribution of LRT For testing, H0 : θ = θ0 vs. H1 : θ = θ1,−2 logλ(x) d→ χ2

1 under regularity condition.Wald Test If Wn is a consistent estimator of θ, and S2

n is a consistentestimator of Var(Wn), then Zn = (Wn − θ0)/Sn follows astandard normal distribution

• Two-sided test : |Zn| > zα/2• One-sided test : Zn > zα/2 or Zn < −zα/2

p-Value A p-value 0 ≤ p(x) ≤ 1 is valid if, Pr(p(X) ≤ α|θ) ≤ α forevery θ ∈ Ω0 and 0 ≤ α ≤ 1.

Constructing p-Value Theorem 8.3.27 : If large W(X) value gives evidencethat H1 is true, p(x) = supθ∈Ω0

Pr(W(X) ≥ W(x)|θ) is avalid p-value

p-Value given sufficient statistics For a sufficient statistic S(X),p(x) = Pr(W(X) ≥ W(x)|S(X) = S(x)) is also a validp-value.

Hyun Min Kang Biostatistics 602 - Lecture 26 Apil 23rd, 2013 8 / 31

Page 46: Biostatistics 602 - Statistical Inference Lecture 26 Final ... · Biostatistics 602 - Statistical Inference Lecture 26 Final Exam Review & Practice Problems for the Final Hyun Min

..........

.....

......

.....

.....

.....

......

.....

.....

.....

......

.....

.....

.....

......

.....

......

.....

.....

.

. . . . . . . .Review

. . . .P1

. . . . . .P2

. . . . .P3

. . . . . .P4

.Wrap-up

Interval Estimation

Coverage probability Pr(θ ∈ [L(X),U(X)])

Coverage coefficient is 1− α if infθ∈Ω Pr(θ ∈ [L(X),U(X)]) = 1− α

Confidence interval [L(X),U(X)]) is 1− α ifinfθ∈Ω Pr(θ ∈ [L(X),U(X)]) = 1− α

Inverting a level α test If A(θ0) is the acceptance region of a level α test,then C(X) = θ : X ∈ A(θ) is a 1− α confidence set (orinterval).

Hyun Min Kang Biostatistics 602 - Lecture 26 Apil 23rd, 2013 9 / 31

Page 47: Biostatistics 602 - Statistical Inference Lecture 26 Final ... · Biostatistics 602 - Statistical Inference Lecture 26 Final Exam Review & Practice Problems for the Final Hyun Min

..........

.....

......

.....

.....

.....

......

.....

.....

.....

......

.....

.....

.....

......

.....

......

.....

.....

.

. . . . . . . .Review

. . . .P1

. . . . . .P2

. . . . .P3

. . . . . .P4

.Wrap-up

Interval Estimation

Coverage probability Pr(θ ∈ [L(X),U(X)])

Coverage coefficient is 1− α if infθ∈Ω Pr(θ ∈ [L(X),U(X)]) = 1− α

Confidence interval [L(X),U(X)]) is 1− α ifinfθ∈Ω Pr(θ ∈ [L(X),U(X)]) = 1− α

Inverting a level α test If A(θ0) is the acceptance region of a level α test,then C(X) = θ : X ∈ A(θ) is a 1− α confidence set (orinterval).

Hyun Min Kang Biostatistics 602 - Lecture 26 Apil 23rd, 2013 9 / 31

Page 48: Biostatistics 602 - Statistical Inference Lecture 26 Final ... · Biostatistics 602 - Statistical Inference Lecture 26 Final Exam Review & Practice Problems for the Final Hyun Min

..........

.....

......

.....

.....

.....

......

.....

.....

.....

......

.....

.....

.....

......

.....

......

.....

.....

.

. . . . . . . .Review

. . . .P1

. . . . . .P2

. . . . .P3

. . . . . .P4

.Wrap-up

Interval Estimation

Coverage probability Pr(θ ∈ [L(X),U(X)])

Coverage coefficient is 1− α if infθ∈Ω Pr(θ ∈ [L(X),U(X)]) = 1− α

Confidence interval [L(X),U(X)]) is 1− α ifinfθ∈Ω Pr(θ ∈ [L(X),U(X)]) = 1− α

Inverting a level α test If A(θ0) is the acceptance region of a level α test,then C(X) = θ : X ∈ A(θ) is a 1− α confidence set (orinterval).

Hyun Min Kang Biostatistics 602 - Lecture 26 Apil 23rd, 2013 9 / 31

Page 49: Biostatistics 602 - Statistical Inference Lecture 26 Final ... · Biostatistics 602 - Statistical Inference Lecture 26 Final Exam Review & Practice Problems for the Final Hyun Min

..........

.....

......

.....

.....

.....

......

.....

.....

.....

......

.....

.....

.....

......

.....

......

.....

.....

.

. . . . . . . .Review

. . . .P1

. . . . . .P2

. . . . .P3

. . . . . .P4

.Wrap-up

Interval Estimation

Coverage probability Pr(θ ∈ [L(X),U(X)])

Coverage coefficient is 1− α if infθ∈Ω Pr(θ ∈ [L(X),U(X)]) = 1− α

Confidence interval [L(X),U(X)]) is 1− α ifinfθ∈Ω Pr(θ ∈ [L(X),U(X)]) = 1− α

Inverting a level α test If A(θ0) is the acceptance region of a level α test,then C(X) = θ : X ∈ A(θ) is a 1− α confidence set (orinterval).

Hyun Min Kang Biostatistics 602 - Lecture 26 Apil 23rd, 2013 9 / 31

Page 50: Biostatistics 602 - Statistical Inference Lecture 26 Final ... · Biostatistics 602 - Statistical Inference Lecture 26 Final Exam Review & Practice Problems for the Final Hyun Min

..........

.....

......

.....

.....

.....

......

.....

.....

.....

......

.....

.....

.....

......

.....

......

.....

.....

.

. . . . . . . .Review

. . . .P1

. . . . . .P2

. . . . .P3

. . . . . .P4

.Wrap-up

Practice Problem 1 (continued from last week)

.Problem..

......

Let f(x|θ) be the logistic location pdf

f(x|θ) =e(x−θ)

(1 + e(x−θ))2−∞ < x < ∞, −∞ < θ < ∞

(a) Show that this family has an MLR(b) Based on one observation X, find the most powerful size α test of

H0 : θ = 0 versus H1 : θ = 1.(c) Show that the test in part (b) is UMP size α for testing H0 : θ ≤ 0 vs.

H1 : θ > 0.

Hyun Min Kang Biostatistics 602 - Lecture 26 Apil 23rd, 2013 10 / 31

Page 51: Biostatistics 602 - Statistical Inference Lecture 26 Final ... · Biostatistics 602 - Statistical Inference Lecture 26 Final Exam Review & Practice Problems for the Final Hyun Min

..........

.....

......

.....

.....

.....

......

.....

.....

.....

......

.....

.....

.....

......

.....

......

.....

.....

.

. . . . . . . .Review

. . . .P1

. . . . . .P2

. . . . .P3

. . . . . .P4

.Wrap-up

Practice Problem 1 (continued from last week)

.Problem..

......

Let f(x|θ) be the logistic location pdf

f(x|θ) =e(x−θ)

(1 + e(x−θ))2−∞ < x < ∞, −∞ < θ < ∞

(a) Show that this family has an MLR

(b) Based on one observation X, find the most powerful size α test ofH0 : θ = 0 versus H1 : θ = 1.

(c) Show that the test in part (b) is UMP size α for testing H0 : θ ≤ 0 vs.H1 : θ > 0.

Hyun Min Kang Biostatistics 602 - Lecture 26 Apil 23rd, 2013 10 / 31

Page 52: Biostatistics 602 - Statistical Inference Lecture 26 Final ... · Biostatistics 602 - Statistical Inference Lecture 26 Final Exam Review & Practice Problems for the Final Hyun Min

..........

.....

......

.....

.....

.....

......

.....

.....

.....

......

.....

.....

.....

......

.....

......

.....

.....

.

. . . . . . . .Review

. . . .P1

. . . . . .P2

. . . . .P3

. . . . . .P4

.Wrap-up

Practice Problem 1 (continued from last week)

.Problem..

......

Let f(x|θ) be the logistic location pdf

f(x|θ) =e(x−θ)

(1 + e(x−θ))2−∞ < x < ∞, −∞ < θ < ∞

(a) Show that this family has an MLR(b) Based on one observation X, find the most powerful size α test of

H0 : θ = 0 versus H1 : θ = 1.

(c) Show that the test in part (b) is UMP size α for testing H0 : θ ≤ 0 vs.H1 : θ > 0.

Hyun Min Kang Biostatistics 602 - Lecture 26 Apil 23rd, 2013 10 / 31

Page 53: Biostatistics 602 - Statistical Inference Lecture 26 Final ... · Biostatistics 602 - Statistical Inference Lecture 26 Final Exam Review & Practice Problems for the Final Hyun Min

..........

.....

......

.....

.....

.....

......

.....

.....

.....

......

.....

.....

.....

......

.....

......

.....

.....

.

. . . . . . . .Review

. . . .P1

. . . . . .P2

. . . . .P3

. . . . . .P4

.Wrap-up

Practice Problem 1 (continued from last week)

.Problem..

......

Let f(x|θ) be the logistic location pdf

f(x|θ) =e(x−θ)

(1 + e(x−θ))2−∞ < x < ∞, −∞ < θ < ∞

(a) Show that this family has an MLR(b) Based on one observation X, find the most powerful size α test of

H0 : θ = 0 versus H1 : θ = 1.(c) Show that the test in part (b) is UMP size α for testing H0 : θ ≤ 0 vs.

H1 : θ > 0.

Hyun Min Kang Biostatistics 602 - Lecture 26 Apil 23rd, 2013 10 / 31

Page 54: Biostatistics 602 - Statistical Inference Lecture 26 Final ... · Biostatistics 602 - Statistical Inference Lecture 26 Final Exam Review & Practice Problems for the Final Hyun Min

..........

.....

......

.....

.....

.....

......

.....

.....

.....

......

.....

.....

.....

......

.....

......

.....

.....

.

. . . . . . . .Review

. . . .P1

. . . . . .P2

. . . . .P3

. . . . . .P4

.Wrap-up

Solution for (a)For θ1 < θ2,

f(x|θ2)f(x|θ1)

=

e(x−θ2)

(1+e(x−θ2))2

e(x−θ1)

(1+e(x−θ1))2

= e(θ1−θ2)

(1 + e(x−θ1)

1 + e(x−θ2)

)2

Let r(x) = (1 + ex−θ1)/(1 + ex−θ2)

r′(x) =e(x−θ1)(1 + e(x−θ2))− (1 + e(x−θ1))e(x−θ2)

(1 + e(x−θ2))2

=e(x−θ1) − e(x−θ2)

(1 + e(x−θ2))2> 0 (∵ x − θ1 > x − θ2)

Therefore, the family of X has an MLR.

Hyun Min Kang Biostatistics 602 - Lecture 26 Apil 23rd, 2013 11 / 31

Page 55: Biostatistics 602 - Statistical Inference Lecture 26 Final ... · Biostatistics 602 - Statistical Inference Lecture 26 Final Exam Review & Practice Problems for the Final Hyun Min

..........

.....

......

.....

.....

.....

......

.....

.....

.....

......

.....

.....

.....

......

.....

......

.....

.....

.

. . . . . . . .Review

. . . .P1

. . . . . .P2

. . . . .P3

. . . . . .P4

.Wrap-up

Solution for (a)For θ1 < θ2,

f(x|θ2)f(x|θ1)

=

e(x−θ2)

(1+e(x−θ2))2

e(x−θ1)

(1+e(x−θ1))2

= e(θ1−θ2)

(1 + e(x−θ1)

1 + e(x−θ2)

)2

Let r(x) = (1 + ex−θ1)/(1 + ex−θ2)

r′(x) =e(x−θ1)(1 + e(x−θ2))− (1 + e(x−θ1))e(x−θ2)

(1 + e(x−θ2))2

=e(x−θ1) − e(x−θ2)

(1 + e(x−θ2))2> 0 (∵ x − θ1 > x − θ2)

Therefore, the family of X has an MLR.

Hyun Min Kang Biostatistics 602 - Lecture 26 Apil 23rd, 2013 11 / 31

Page 56: Biostatistics 602 - Statistical Inference Lecture 26 Final ... · Biostatistics 602 - Statistical Inference Lecture 26 Final Exam Review & Practice Problems for the Final Hyun Min

..........

.....

......

.....

.....

.....

......

.....

.....

.....

......

.....

.....

.....

......

.....

......

.....

.....

.

. . . . . . . .Review

. . . .P1

. . . . . .P2

. . . . .P3

. . . . . .P4

.Wrap-up

Solution for (a)For θ1 < θ2,

f(x|θ2)f(x|θ1)

=

e(x−θ2)

(1+e(x−θ2))2

e(x−θ1)

(1+e(x−θ1))2

= e(θ1−θ2)

(1 + e(x−θ1)

1 + e(x−θ2)

)2

Let r(x) = (1 + ex−θ1)/(1 + ex−θ2)

r′(x) =e(x−θ1)(1 + e(x−θ2))− (1 + e(x−θ1))e(x−θ2)

(1 + e(x−θ2))2

=e(x−θ1) − e(x−θ2)

(1 + e(x−θ2))2> 0 (∵ x − θ1 > x − θ2)

Therefore, the family of X has an MLR.

Hyun Min Kang Biostatistics 602 - Lecture 26 Apil 23rd, 2013 11 / 31

Page 57: Biostatistics 602 - Statistical Inference Lecture 26 Final ... · Biostatistics 602 - Statistical Inference Lecture 26 Final Exam Review & Practice Problems for the Final Hyun Min

..........

.....

......

.....

.....

.....

......

.....

.....

.....

......

.....

.....

.....

......

.....

......

.....

.....

.

. . . . . . . .Review

. . . .P1

. . . . . .P2

. . . . .P3

. . . . . .P4

.Wrap-up

Solution for (a)For θ1 < θ2,

f(x|θ2)f(x|θ1)

=

e(x−θ2)

(1+e(x−θ2))2

e(x−θ1)

(1+e(x−θ1))2

= e(θ1−θ2)

(1 + e(x−θ1)

1 + e(x−θ2)

)2

Let r(x) = (1 + ex−θ1)/(1 + ex−θ2)

r′(x) =e(x−θ1)(1 + e(x−θ2))− (1 + e(x−θ1))e(x−θ2)

(1 + e(x−θ2))2

=e(x−θ1) − e(x−θ2)

(1 + e(x−θ2))2> 0 (∵ x − θ1 > x − θ2)

Therefore, the family of X has an MLR.

Hyun Min Kang Biostatistics 602 - Lecture 26 Apil 23rd, 2013 11 / 31

Page 58: Biostatistics 602 - Statistical Inference Lecture 26 Final ... · Biostatistics 602 - Statistical Inference Lecture 26 Final Exam Review & Practice Problems for the Final Hyun Min

..........

.....

......

.....

.....

.....

......

.....

.....

.....

......

.....

.....

.....

......

.....

......

.....

.....

.

. . . . . . . .Review

. . . .P1

. . . . . .P2

. . . . .P3

. . . . . .P4

.Wrap-up

Solution for (a)For θ1 < θ2,

f(x|θ2)f(x|θ1)

=

e(x−θ2)

(1+e(x−θ2))2

e(x−θ1)

(1+e(x−θ1))2

= e(θ1−θ2)

(1 + e(x−θ1)

1 + e(x−θ2)

)2

Let r(x) = (1 + ex−θ1)/(1 + ex−θ2)

r′(x) =e(x−θ1)(1 + e(x−θ2))− (1 + e(x−θ1))e(x−θ2)

(1 + e(x−θ2))2

=e(x−θ1) − e(x−θ2)

(1 + e(x−θ2))2> 0 (∵ x − θ1 > x − θ2)

Therefore, the family of X has an MLR.

Hyun Min Kang Biostatistics 602 - Lecture 26 Apil 23rd, 2013 11 / 31

Page 59: Biostatistics 602 - Statistical Inference Lecture 26 Final ... · Biostatistics 602 - Statistical Inference Lecture 26 Final Exam Review & Practice Problems for the Final Hyun Min

..........

.....

......

.....

.....

.....

......

.....

.....

.....

......

.....

.....

.....

......

.....

......

.....

.....

.

. . . . . . . .Review

. . . .P1

. . . . . .P2

. . . . .P3

. . . . . .P4

.Wrap-up

Solution for (a)For θ1 < θ2,

f(x|θ2)f(x|θ1)

=

e(x−θ2)

(1+e(x−θ2))2

e(x−θ1)

(1+e(x−θ1))2

= e(θ1−θ2)

(1 + e(x−θ1)

1 + e(x−θ2)

)2

Let r(x) = (1 + ex−θ1)/(1 + ex−θ2)

r′(x) =e(x−θ1)(1 + e(x−θ2))− (1 + e(x−θ1))e(x−θ2)

(1 + e(x−θ2))2

=e(x−θ1) − e(x−θ2)

(1 + e(x−θ2))2> 0 (∵ x − θ1 > x − θ2)

Therefore, the family of X has an MLR.Hyun Min Kang Biostatistics 602 - Lecture 26 Apil 23rd, 2013 11 / 31

Page 60: Biostatistics 602 - Statistical Inference Lecture 26 Final ... · Biostatistics 602 - Statistical Inference Lecture 26 Final Exam Review & Practice Problems for the Final Hyun Min

..........

.....

......

.....

.....

.....

......

.....

.....

.....

......

.....

.....

.....

......

.....

......

.....

.....

.

. . . . . . . .Review

. . . .P1

. . . . . .P2

. . . . .P3

. . . . . .P4

.Wrap-up

Solution for (b)The UMP test rejects H0 if and only if

f(x|1)f(x|0) = e

(1 + ex

1 + e(x−1)

)2

> k

1 + ex

1 + e(x−1)> k∗

1 + ex

e + ex > k∗∗

X > x0Because under H0, F(x0|θ = 0) = ex

1+ex , the rejection region of UMP levelα test satisfies

1− F(x|θ = 0) =1

1 + ex0 = α

x0 ∼ log(1− α

α

)

Hyun Min Kang Biostatistics 602 - Lecture 26 Apil 23rd, 2013 12 / 31

Page 61: Biostatistics 602 - Statistical Inference Lecture 26 Final ... · Biostatistics 602 - Statistical Inference Lecture 26 Final Exam Review & Practice Problems for the Final Hyun Min

..........

.....

......

.....

.....

.....

......

.....

.....

.....

......

.....

.....

.....

......

.....

......

.....

.....

.

. . . . . . . .Review

. . . .P1

. . . . . .P2

. . . . .P3

. . . . . .P4

.Wrap-up

Solution for (b)The UMP test rejects H0 if and only if

f(x|1)f(x|0) = e

(1 + ex

1 + e(x−1)

)2

> k

1 + ex

1 + e(x−1)> k∗

1 + ex

e + ex > k∗∗

X > x0Because under H0, F(x0|θ = 0) = ex

1+ex , the rejection region of UMP levelα test satisfies

1− F(x|θ = 0) =1

1 + ex0 = α

x0 ∼ log(1− α

α

)

Hyun Min Kang Biostatistics 602 - Lecture 26 Apil 23rd, 2013 12 / 31

Page 62: Biostatistics 602 - Statistical Inference Lecture 26 Final ... · Biostatistics 602 - Statistical Inference Lecture 26 Final Exam Review & Practice Problems for the Final Hyun Min

..........

.....

......

.....

.....

.....

......

.....

.....

.....

......

.....

.....

.....

......

.....

......

.....

.....

.

. . . . . . . .Review

. . . .P1

. . . . . .P2

. . . . .P3

. . . . . .P4

.Wrap-up

Solution for (b)The UMP test rejects H0 if and only if

f(x|1)f(x|0) = e

(1 + ex

1 + e(x−1)

)2

> k

1 + ex

1 + e(x−1)> k∗

1 + ex

e + ex > k∗∗

X > x0Because under H0, F(x0|θ = 0) = ex

1+ex , the rejection region of UMP levelα test satisfies

1− F(x|θ = 0) =1

1 + ex0 = α

x0 ∼ log(1− α

α

)

Hyun Min Kang Biostatistics 602 - Lecture 26 Apil 23rd, 2013 12 / 31

Page 63: Biostatistics 602 - Statistical Inference Lecture 26 Final ... · Biostatistics 602 - Statistical Inference Lecture 26 Final Exam Review & Practice Problems for the Final Hyun Min

..........

.....

......

.....

.....

.....

......

.....

.....

.....

......

.....

.....

.....

......

.....

......

.....

.....

.

. . . . . . . .Review

. . . .P1

. . . . . .P2

. . . . .P3

. . . . . .P4

.Wrap-up

Solution for (b)The UMP test rejects H0 if and only if

f(x|1)f(x|0) = e

(1 + ex

1 + e(x−1)

)2

> k

1 + ex

1 + e(x−1)> k∗

1 + ex

e + ex > k∗∗

X > x0

Because under H0, F(x0|θ = 0) = ex

1+ex , the rejection region of UMP levelα test satisfies

1− F(x|θ = 0) =1

1 + ex0 = α

x0 ∼ log(1− α

α

)

Hyun Min Kang Biostatistics 602 - Lecture 26 Apil 23rd, 2013 12 / 31

Page 64: Biostatistics 602 - Statistical Inference Lecture 26 Final ... · Biostatistics 602 - Statistical Inference Lecture 26 Final Exam Review & Practice Problems for the Final Hyun Min

..........

.....

......

.....

.....

.....

......

.....

.....

.....

......

.....

.....

.....

......

.....

......

.....

.....

.

. . . . . . . .Review

. . . .P1

. . . . . .P2

. . . . .P3

. . . . . .P4

.Wrap-up

Solution for (b)The UMP test rejects H0 if and only if

f(x|1)f(x|0) = e

(1 + ex

1 + e(x−1)

)2

> k

1 + ex

1 + e(x−1)> k∗

1 + ex

e + ex > k∗∗

X > x0Because under H0, F(x0|θ = 0) = ex

1+ex , the rejection region of UMP levelα test satisfies

1− F(x|θ = 0) =1

1 + ex0 = α

x0 ∼ log(1− α

α

)Hyun Min Kang Biostatistics 602 - Lecture 26 Apil 23rd, 2013 12 / 31

Page 65: Biostatistics 602 - Statistical Inference Lecture 26 Final ... · Biostatistics 602 - Statistical Inference Lecture 26 Final Exam Review & Practice Problems for the Final Hyun Min

..........

.....

......

.....

.....

.....

......

.....

.....

.....

......

.....

.....

.....

......

.....

......

.....

.....

.

. . . . . . . .Review

. . . .P1

. . . . . .P2

. . . . .P3

. . . . . .P4

.Wrap-up

Solution for (c)

Because the family of X has an MLR, UMP size α for testing H0 : θ ≤ 0vs. H1 : θ > 0 should be a form of

X > x0Pr(X > x0|θ = 0) = α

Therefore, x0 = log(1−αα

), which is identical to the test defined in (b).

Hyun Min Kang Biostatistics 602 - Lecture 26 Apil 23rd, 2013 13 / 31

Page 66: Biostatistics 602 - Statistical Inference Lecture 26 Final ... · Biostatistics 602 - Statistical Inference Lecture 26 Final Exam Review & Practice Problems for the Final Hyun Min

..........

.....

......

.....

.....

.....

......

.....

.....

.....

......

.....

.....

.....

......

.....

......

.....

.....

.

. . . . . . . .Review

. . . .P1

. . . . . .P2

. . . . .P3

. . . . . .P4

.Wrap-up

Solution for (c)

Because the family of X has an MLR, UMP size α for testing H0 : θ ≤ 0vs. H1 : θ > 0 should be a form of

X > x0Pr(X > x0|θ = 0) = α

Therefore, x0 = log(1−αα

), which is identical to the test defined in (b).

Hyun Min Kang Biostatistics 602 - Lecture 26 Apil 23rd, 2013 13 / 31

Page 67: Biostatistics 602 - Statistical Inference Lecture 26 Final ... · Biostatistics 602 - Statistical Inference Lecture 26 Final Exam Review & Practice Problems for the Final Hyun Min

..........

.....

......

.....

.....

.....

......

.....

.....

.....

......

.....

.....

.....

......

.....

......

.....

.....

.

. . . . . . . .Review

. . . .P1

. . . . . .P2

. . . . .P3

. . . . . .P4

.Wrap-up

Practice Problem 2

.Problem..

......

Suppose X1, · · · ,Xn are iid random samples with pdffX(x|θ) = θ exp(−θx), where x ≥ 0, θ > 0

(a) Show that n∑nx=1 Xi

is a consistent estimator for θ.(b) Show that n∑n

x=1 Xiis asymptotically normal and derive its asymptotic

distribution(c) Derive the Wald asymptotic size α test for H0 : θ = θ0 vs. H1 : θ = θ0.(d) Find an asymptotic (1− α) confidence interval for θ by inverting the

above testYou may use the fact that EX = 1/θ and Var(X) = 1/θ2.

Hyun Min Kang Biostatistics 602 - Lecture 26 Apil 23rd, 2013 14 / 31

Page 68: Biostatistics 602 - Statistical Inference Lecture 26 Final ... · Biostatistics 602 - Statistical Inference Lecture 26 Final Exam Review & Practice Problems for the Final Hyun Min

..........

.....

......

.....

.....

.....

......

.....

.....

.....

......

.....

.....

.....

......

.....

......

.....

.....

.

. . . . . . . .Review

. . . .P1

. . . . . .P2

. . . . .P3

. . . . . .P4

.Wrap-up

Practice Problem 2

.Problem..

......

Suppose X1, · · · ,Xn are iid random samples with pdffX(x|θ) = θ exp(−θx), where x ≥ 0, θ > 0

(a) Show that n∑nx=1 Xi

is a consistent estimator for θ.

(b) Show that n∑nx=1 Xi

is asymptotically normal and derive its asymptoticdistribution

(c) Derive the Wald asymptotic size α test for H0 : θ = θ0 vs. H1 : θ = θ0.(d) Find an asymptotic (1− α) confidence interval for θ by inverting the

above testYou may use the fact that EX = 1/θ and Var(X) = 1/θ2.

Hyun Min Kang Biostatistics 602 - Lecture 26 Apil 23rd, 2013 14 / 31

Page 69: Biostatistics 602 - Statistical Inference Lecture 26 Final ... · Biostatistics 602 - Statistical Inference Lecture 26 Final Exam Review & Practice Problems for the Final Hyun Min

..........

.....

......

.....

.....

.....

......

.....

.....

.....

......

.....

.....

.....

......

.....

......

.....

.....

.

. . . . . . . .Review

. . . .P1

. . . . . .P2

. . . . .P3

. . . . . .P4

.Wrap-up

Practice Problem 2

.Problem..

......

Suppose X1, · · · ,Xn are iid random samples with pdffX(x|θ) = θ exp(−θx), where x ≥ 0, θ > 0

(a) Show that n∑nx=1 Xi

is a consistent estimator for θ.(b) Show that n∑n

x=1 Xiis asymptotically normal and derive its asymptotic

distribution

(c) Derive the Wald asymptotic size α test for H0 : θ = θ0 vs. H1 : θ = θ0.(d) Find an asymptotic (1− α) confidence interval for θ by inverting the

above testYou may use the fact that EX = 1/θ and Var(X) = 1/θ2.

Hyun Min Kang Biostatistics 602 - Lecture 26 Apil 23rd, 2013 14 / 31

Page 70: Biostatistics 602 - Statistical Inference Lecture 26 Final ... · Biostatistics 602 - Statistical Inference Lecture 26 Final Exam Review & Practice Problems for the Final Hyun Min

..........

.....

......

.....

.....

.....

......

.....

.....

.....

......

.....

.....

.....

......

.....

......

.....

.....

.

. . . . . . . .Review

. . . .P1

. . . . . .P2

. . . . .P3

. . . . . .P4

.Wrap-up

Practice Problem 2

.Problem..

......

Suppose X1, · · · ,Xn are iid random samples with pdffX(x|θ) = θ exp(−θx), where x ≥ 0, θ > 0

(a) Show that n∑nx=1 Xi

is a consistent estimator for θ.(b) Show that n∑n

x=1 Xiis asymptotically normal and derive its asymptotic

distribution(c) Derive the Wald asymptotic size α test for H0 : θ = θ0 vs. H1 : θ = θ0.

(d) Find an asymptotic (1− α) confidence interval for θ by inverting theabove test

You may use the fact that EX = 1/θ and Var(X) = 1/θ2.

Hyun Min Kang Biostatistics 602 - Lecture 26 Apil 23rd, 2013 14 / 31

Page 71: Biostatistics 602 - Statistical Inference Lecture 26 Final ... · Biostatistics 602 - Statistical Inference Lecture 26 Final Exam Review & Practice Problems for the Final Hyun Min

..........

.....

......

.....

.....

.....

......

.....

.....

.....

......

.....

.....

.....

......

.....

......

.....

.....

.

. . . . . . . .Review

. . . .P1

. . . . . .P2

. . . . .P3

. . . . . .P4

.Wrap-up

Practice Problem 2

.Problem..

......

Suppose X1, · · · ,Xn are iid random samples with pdffX(x|θ) = θ exp(−θx), where x ≥ 0, θ > 0

(a) Show that n∑nx=1 Xi

is a consistent estimator for θ.(b) Show that n∑n

x=1 Xiis asymptotically normal and derive its asymptotic

distribution(c) Derive the Wald asymptotic size α test for H0 : θ = θ0 vs. H1 : θ = θ0.(d) Find an asymptotic (1− α) confidence interval for θ by inverting the

above testYou may use the fact that EX = 1/θ and Var(X) = 1/θ2.

Hyun Min Kang Biostatistics 602 - Lecture 26 Apil 23rd, 2013 14 / 31

Page 72: Biostatistics 602 - Statistical Inference Lecture 26 Final ... · Biostatistics 602 - Statistical Inference Lecture 26 Final Exam Review & Practice Problems for the Final Hyun Min

..........

.....

......

.....

.....

.....

......

.....

.....

.....

......

.....

.....

.....

......

.....

......

.....

.....

.

. . . . . . . .Review

. . . .P1

. . . . . .P2

. . . . .P3

. . . . . .P4

.Wrap-up

Solution (a) - Consistency

..1 Obtain EX = 1/θ (Derive yourself if not given)

EX =

∫ ∞

0xf(x|θ)dx =

∫ ∞

0θx exp(−θx)dx

= [−x exp(−θx)]∞0 +

∫ ∞

0exp(−θx)dx

= 0 +

[−1

θexp(−θx)

]∞0

=1

θ

..2 By LLN (Law of Large Number), X P→ EX = 1/θ.

..3 By Theorem of continuous map, n/∑n

i=1 Xi = 1/X P→ θ.

Hyun Min Kang Biostatistics 602 - Lecture 26 Apil 23rd, 2013 15 / 31

Page 73: Biostatistics 602 - Statistical Inference Lecture 26 Final ... · Biostatistics 602 - Statistical Inference Lecture 26 Final Exam Review & Practice Problems for the Final Hyun Min

..........

.....

......

.....

.....

.....

......

.....

.....

.....

......

.....

.....

.....

......

.....

......

.....

.....

.

. . . . . . . .Review

. . . .P1

. . . . . .P2

. . . . .P3

. . . . . .P4

.Wrap-up

Solution (a) - Consistency

..1 Obtain EX = 1/θ (Derive yourself if not given)

EX =

∫ ∞

0xf(x|θ)dx =

∫ ∞

0θx exp(−θx)dx

= [−x exp(−θx)]∞0 +

∫ ∞

0exp(−θx)dx

= 0 +

[−1

θexp(−θx)

]∞0

=1

θ

..2 By LLN (Law of Large Number), X P→ EX = 1/θ.

..3 By Theorem of continuous map, n/∑n

i=1 Xi = 1/X P→ θ.

Hyun Min Kang Biostatistics 602 - Lecture 26 Apil 23rd, 2013 15 / 31

Page 74: Biostatistics 602 - Statistical Inference Lecture 26 Final ... · Biostatistics 602 - Statistical Inference Lecture 26 Final Exam Review & Practice Problems for the Final Hyun Min

..........

.....

......

.....

.....

.....

......

.....

.....

.....

......

.....

.....

.....

......

.....

......

.....

.....

.

. . . . . . . .Review

. . . .P1

. . . . . .P2

. . . . .P3

. . . . . .P4

.Wrap-up

Solution (a) - Consistency

..1 Obtain EX = 1/θ (Derive yourself if not given)

EX =

∫ ∞

0xf(x|θ)dx =

∫ ∞

0θx exp(−θx)dx

= [−x exp(−θx)]∞0 +

∫ ∞

0exp(−θx)dx

= 0 +

[−1

θexp(−θx)

]∞0

=1

θ

..2 By LLN (Law of Large Number), X P→ EX = 1/θ.

..3 By Theorem of continuous map, n/∑n

i=1 Xi = 1/X P→ θ.

Hyun Min Kang Biostatistics 602 - Lecture 26 Apil 23rd, 2013 15 / 31

Page 75: Biostatistics 602 - Statistical Inference Lecture 26 Final ... · Biostatistics 602 - Statistical Inference Lecture 26 Final Exam Review & Practice Problems for the Final Hyun Min

..........

.....

......

.....

.....

.....

......

.....

.....

.....

......

.....

.....

.....

......

.....

......

.....

.....

.

. . . . . . . .Review

. . . .P1

. . . . . .P2

. . . . .P3

. . . . . .P4

.Wrap-up

Solution (a) - Consistency

..1 Obtain EX = 1/θ (Derive yourself if not given)

EX =

∫ ∞

0xf(x|θ)dx =

∫ ∞

0θx exp(−θx)dx

= [−x exp(−θx)]∞0 +

∫ ∞

0exp(−θx)dx

= 0 +

[−1

θexp(−θx)

]∞0

=1

θ

..2 By LLN (Law of Large Number), X P→ EX = 1/θ.

..3 By Theorem of continuous map, n/∑n

i=1 Xi = 1/X P→ θ.

Hyun Min Kang Biostatistics 602 - Lecture 26 Apil 23rd, 2013 15 / 31

Page 76: Biostatistics 602 - Statistical Inference Lecture 26 Final ... · Biostatistics 602 - Statistical Inference Lecture 26 Final Exam Review & Practice Problems for the Final Hyun Min

..........

.....

......

.....

.....

.....

......

.....

.....

.....

......

.....

.....

.....

......

.....

......

.....

.....

.

. . . . . . . .Review

. . . .P1

. . . . . .P2

. . . . .P3

. . . . . .P4

.Wrap-up

Solution (a) - Consistency

..1 Obtain EX = 1/θ (Derive yourself if not given)

EX =

∫ ∞

0xf(x|θ)dx =

∫ ∞

0θx exp(−θx)dx

= [−x exp(−θx)]∞0 +

∫ ∞

0exp(−θx)dx

= 0 +

[−1

θexp(−θx)

]∞0

=1

θ

..2 By LLN (Law of Large Number), X P→ EX = 1/θ.

..3 By Theorem of continuous map, n/∑n

i=1 Xi = 1/X P→ θ.

Hyun Min Kang Biostatistics 602 - Lecture 26 Apil 23rd, 2013 15 / 31

Page 77: Biostatistics 602 - Statistical Inference Lecture 26 Final ... · Biostatistics 602 - Statistical Inference Lecture 26 Final Exam Review & Practice Problems for the Final Hyun Min

..........

.....

......

.....

.....

.....

......

.....

.....

.....

......

.....

.....

.....

......

.....

......

.....

.....

.

. . . . . . . .Review

. . . .P1

. . . . . .P2

. . . . .P3

. . . . . .P4

.Wrap-up

Solution (b) - Asymptotic Distribution

..1 Obtain Var(X) = 1/θ2 (Derive if needed, omitted here).

..2 Apply CLT(Central Limit Theorem),

X ∼ AN(1

θ,

1

θ2n

)

..3 Apply Delta method. Let g(y) = 1/y, then g′(y) = −1/y2.∑Xi

n = 1/X = g(X) ∼ AN(

g(1/θ), [g′(1/θ)]2

θ2n

)= AN

(θ,

θ2

n

)⇐⇒

√n(1

X− θ

)= N

(0, θ2

)

Hyun Min Kang Biostatistics 602 - Lecture 26 Apil 23rd, 2013 16 / 31

Page 78: Biostatistics 602 - Statistical Inference Lecture 26 Final ... · Biostatistics 602 - Statistical Inference Lecture 26 Final Exam Review & Practice Problems for the Final Hyun Min

..........

.....

......

.....

.....

.....

......

.....

.....

.....

......

.....

.....

.....

......

.....

......

.....

.....

.

. . . . . . . .Review

. . . .P1

. . . . . .P2

. . . . .P3

. . . . . .P4

.Wrap-up

Solution (b) - Asymptotic Distribution

..1 Obtain Var(X) = 1/θ2 (Derive if needed, omitted here).

..2 Apply CLT(Central Limit Theorem),

X ∼ AN(1

θ,

1

θ2n

)

..3 Apply Delta method. Let g(y) = 1/y, then g′(y) = −1/y2.∑Xi

n = 1/X = g(X) ∼ AN(

g(1/θ), [g′(1/θ)]2

θ2n

)= AN

(θ,

θ2

n

)⇐⇒

√n(1

X− θ

)= N

(0, θ2

)

Hyun Min Kang Biostatistics 602 - Lecture 26 Apil 23rd, 2013 16 / 31

Page 79: Biostatistics 602 - Statistical Inference Lecture 26 Final ... · Biostatistics 602 - Statistical Inference Lecture 26 Final Exam Review & Practice Problems for the Final Hyun Min

..........

.....

......

.....

.....

.....

......

.....

.....

.....

......

.....

.....

.....

......

.....

......

.....

.....

.

. . . . . . . .Review

. . . .P1

. . . . . .P2

. . . . .P3

. . . . . .P4

.Wrap-up

Solution (b) - Asymptotic Distribution

..1 Obtain Var(X) = 1/θ2 (Derive if needed, omitted here).

..2 Apply CLT(Central Limit Theorem),

X ∼ AN(1

θ,

1

θ2n

)

..3 Apply Delta method. Let g(y) = 1/y, then g′(y) = −1/y2.

∑Xi

n = 1/X = g(X) ∼ AN(

g(1/θ), [g′(1/θ)]2

θ2n

)= AN

(θ,

θ2

n

)⇐⇒

√n(1

X− θ

)= N

(0, θ2

)

Hyun Min Kang Biostatistics 602 - Lecture 26 Apil 23rd, 2013 16 / 31

Page 80: Biostatistics 602 - Statistical Inference Lecture 26 Final ... · Biostatistics 602 - Statistical Inference Lecture 26 Final Exam Review & Practice Problems for the Final Hyun Min

..........

.....

......

.....

.....

.....

......

.....

.....

.....

......

.....

.....

.....

......

.....

......

.....

.....

.

. . . . . . . .Review

. . . .P1

. . . . . .P2

. . . . .P3

. . . . . .P4

.Wrap-up

Solution (b) - Asymptotic Distribution

..1 Obtain Var(X) = 1/θ2 (Derive if needed, omitted here).

..2 Apply CLT(Central Limit Theorem),

X ∼ AN(1

θ,

1

θ2n

)

..3 Apply Delta method. Let g(y) = 1/y, then g′(y) = −1/y2.∑Xi

n = 1/X = g(X) ∼ AN(

g(1/θ), [g′(1/θ)]2

θ2n

)

= AN(θ,

θ2

n

)⇐⇒

√n(1

X− θ

)= N

(0, θ2

)

Hyun Min Kang Biostatistics 602 - Lecture 26 Apil 23rd, 2013 16 / 31

Page 81: Biostatistics 602 - Statistical Inference Lecture 26 Final ... · Biostatistics 602 - Statistical Inference Lecture 26 Final Exam Review & Practice Problems for the Final Hyun Min

..........

.....

......

.....

.....

.....

......

.....

.....

.....

......

.....

.....

.....

......

.....

......

.....

.....

.

. . . . . . . .Review

. . . .P1

. . . . . .P2

. . . . .P3

. . . . . .P4

.Wrap-up

Solution (b) - Asymptotic Distribution

..1 Obtain Var(X) = 1/θ2 (Derive if needed, omitted here).

..2 Apply CLT(Central Limit Theorem),

X ∼ AN(1

θ,

1

θ2n

)

..3 Apply Delta method. Let g(y) = 1/y, then g′(y) = −1/y2.∑Xi

n = 1/X = g(X) ∼ AN(

g(1/θ), [g′(1/θ)]2

θ2n

)= AN

(θ,

θ2

n

)

⇐⇒√

n(1

X− θ

)= N

(0, θ2

)

Hyun Min Kang Biostatistics 602 - Lecture 26 Apil 23rd, 2013 16 / 31

Page 82: Biostatistics 602 - Statistical Inference Lecture 26 Final ... · Biostatistics 602 - Statistical Inference Lecture 26 Final Exam Review & Practice Problems for the Final Hyun Min

..........

.....

......

.....

.....

.....

......

.....

.....

.....

......

.....

.....

.....

......

.....

......

.....

.....

.

. . . . . . . .Review

. . . .P1

. . . . . .P2

. . . . .P3

. . . . . .P4

.Wrap-up

Solution (b) - Asymptotic Distribution

..1 Obtain Var(X) = 1/θ2 (Derive if needed, omitted here).

..2 Apply CLT(Central Limit Theorem),

X ∼ AN(1

θ,

1

θ2n

)

..3 Apply Delta method. Let g(y) = 1/y, then g′(y) = −1/y2.∑Xi

n = 1/X = g(X) ∼ AN(

g(1/θ), [g′(1/θ)]2

θ2n

)= AN

(θ,

θ2

n

)⇐⇒

√n(1

X− θ

)= N

(0, θ2

)Hyun Min Kang Biostatistics 602 - Lecture 26 Apil 23rd, 2013 16 / 31

Page 83: Biostatistics 602 - Statistical Inference Lecture 26 Final ... · Biostatistics 602 - Statistical Inference Lecture 26 Final Exam Review & Practice Problems for the Final Hyun Min

..........

.....

......

.....

.....

.....

......

.....

.....

.....

......

.....

.....

.....

......

.....

......

.....

.....

.

. . . . . . . .Review

. . . .P1

. . . . . .P2

. . . . .P3

. . . . . .P4

.Wrap-up

Solution (c) - Wald asymptotic size α test

..1 Obtain a consistent estimator of θ :

W(X) =

∑ni=1 Xin ∼ AN

(θ,

θ2

n

)..2 Obtain a constant estimator of Var(W)

1

n − 1

n∑i=1

(Xi − X)2P→ Var(X) =

1

θ2(CLT)

n − 1∑ni=1(Xi − X)2

P→ θ2 (Continuous Map Theorem).

S2 =n∑n

i=1(Xi − X)2P→ θ2 (Slutsky’s Theorem).

Hyun Min Kang Biostatistics 602 - Lecture 26 Apil 23rd, 2013 17 / 31

Page 84: Biostatistics 602 - Statistical Inference Lecture 26 Final ... · Biostatistics 602 - Statistical Inference Lecture 26 Final Exam Review & Practice Problems for the Final Hyun Min

..........

.....

......

.....

.....

.....

......

.....

.....

.....

......

.....

.....

.....

......

.....

......

.....

.....

.

. . . . . . . .Review

. . . .P1

. . . . . .P2

. . . . .P3

. . . . . .P4

.Wrap-up

Solution (c) - Wald asymptotic size α test

..1 Obtain a consistent estimator of θ :

W(X) =

∑ni=1 Xin ∼ AN

(θ,

θ2

n

)

..2 Obtain a constant estimator of Var(W)

1

n − 1

n∑i=1

(Xi − X)2P→ Var(X) =

1

θ2(CLT)

n − 1∑ni=1(Xi − X)2

P→ θ2 (Continuous Map Theorem).

S2 =n∑n

i=1(Xi − X)2P→ θ2 (Slutsky’s Theorem).

Hyun Min Kang Biostatistics 602 - Lecture 26 Apil 23rd, 2013 17 / 31

Page 85: Biostatistics 602 - Statistical Inference Lecture 26 Final ... · Biostatistics 602 - Statistical Inference Lecture 26 Final Exam Review & Practice Problems for the Final Hyun Min

..........

.....

......

.....

.....

.....

......

.....

.....

.....

......

.....

.....

.....

......

.....

......

.....

.....

.

. . . . . . . .Review

. . . .P1

. . . . . .P2

. . . . .P3

. . . . . .P4

.Wrap-up

Solution (c) - Wald asymptotic size α test

..1 Obtain a consistent estimator of θ :

W(X) =

∑ni=1 Xin ∼ AN

(θ,

θ2

n

)..2 Obtain a constant estimator of Var(W)

1

n − 1

n∑i=1

(Xi − X)2P→ Var(X) =

1

θ2(CLT)

n − 1∑ni=1(Xi − X)2

P→ θ2 (Continuous Map Theorem).

S2 =n∑n

i=1(Xi − X)2P→ θ2 (Slutsky’s Theorem).

Hyun Min Kang Biostatistics 602 - Lecture 26 Apil 23rd, 2013 17 / 31

Page 86: Biostatistics 602 - Statistical Inference Lecture 26 Final ... · Biostatistics 602 - Statistical Inference Lecture 26 Final Exam Review & Practice Problems for the Final Hyun Min

..........

.....

......

.....

.....

.....

......

.....

.....

.....

......

.....

.....

.....

......

.....

......

.....

.....

.

. . . . . . . .Review

. . . .P1

. . . . . .P2

. . . . .P3

. . . . . .P4

.Wrap-up

Solution (c) - Wald asymptotic size α test

..1 Obtain a consistent estimator of θ :

W(X) =

∑ni=1 Xin ∼ AN

(θ,

θ2

n

)..2 Obtain a constant estimator of Var(W)

1

n − 1

n∑i=1

(Xi − X)2P→ Var(X) =

1

θ2(CLT)

n − 1∑ni=1(Xi − X)2

P→ θ2 (Continuous Map Theorem).

S2 =n∑n

i=1(Xi − X)2P→ θ2 (Slutsky’s Theorem).

Hyun Min Kang Biostatistics 602 - Lecture 26 Apil 23rd, 2013 17 / 31

Page 87: Biostatistics 602 - Statistical Inference Lecture 26 Final ... · Biostatistics 602 - Statistical Inference Lecture 26 Final Exam Review & Practice Problems for the Final Hyun Min

..........

.....

......

.....

.....

.....

......

.....

.....

.....

......

.....

.....

.....

......

.....

......

.....

.....

.

. . . . . . . .Review

. . . .P1

. . . . . .P2

. . . . .P3

. . . . . .P4

.Wrap-up

Solution (c) - Wald asymptotic size α test

..1 Obtain a consistent estimator of θ :

W(X) =

∑ni=1 Xin ∼ AN

(θ,

θ2

n

)..2 Obtain a constant estimator of Var(W)

1

n − 1

n∑i=1

(Xi − X)2P→ Var(X) =

1

θ2(CLT)

n − 1∑ni=1(Xi − X)2

P→ θ2 (Continuous Map Theorem).

S2 =n∑n

i=1(Xi − X)2P→ θ2 (Slutsky’s Theorem).

Hyun Min Kang Biostatistics 602 - Lecture 26 Apil 23rd, 2013 17 / 31

Page 88: Biostatistics 602 - Statistical Inference Lecture 26 Final ... · Biostatistics 602 - Statistical Inference Lecture 26 Final Exam Review & Practice Problems for the Final Hyun Min

..........

.....

......

.....

.....

.....

......

.....

.....

.....

......

.....

.....

.....

......

.....

......

.....

.....

.

. . . . . . . .Review

. . . .P1

. . . . . .P2

. . . . .P3

. . . . . .P4

.Wrap-up

Solution (c) - Wald asymptotic size α test

..1 Obtain a consistent estimator of θ :

W(X) =

∑ni=1 Xin ∼ AN

(θ,

θ2

n

)..2 Obtain a constant estimator of Var(W)

1

n − 1

n∑i=1

(Xi − X)2P→ Var(X) =

1

θ2(CLT)

n − 1∑ni=1(Xi − X)2

P→ θ2 (Continuous Map Theorem).

S2 =n∑n

i=1(Xi − X)2P→ θ2 (Slutsky’s Theorem).

Hyun Min Kang Biostatistics 602 - Lecture 26 Apil 23rd, 2013 17 / 31

Page 89: Biostatistics 602 - Statistical Inference Lecture 26 Final ... · Biostatistics 602 - Statistical Inference Lecture 26 Final Exam Review & Practice Problems for the Final Hyun Min

..........

.....

......

.....

.....

.....

......

.....

.....

.....

......

.....

.....

.....

......

.....

......

.....

.....

.

. . . . . . . .Review

. . . .P1

. . . . . .P2

. . . . .P3

. . . . . .P4

.Wrap-up

Solution (c) - Wald Asymptotic size α test (cont’d)

..3 Construct a two-sided asymptotic size α Wald test, whose rejectionregion is

|Z(X)| =

∣∣∣∣W(X)− θ0S/

√n

∣∣∣∣=

∣∣∣∣∣∣n∑n

i=1 Xi− θ0

1√∑ni=1(Xi−X)2

∣∣∣∣∣∣=

∣∣∣∣ 1X − θ0

∣∣∣∣√√√√ n∑

i=1

(Xi − X)2 ≥ zα/2

Hyun Min Kang Biostatistics 602 - Lecture 26 Apil 23rd, 2013 18 / 31

Page 90: Biostatistics 602 - Statistical Inference Lecture 26 Final ... · Biostatistics 602 - Statistical Inference Lecture 26 Final Exam Review & Practice Problems for the Final Hyun Min

..........

.....

......

.....

.....

.....

......

.....

.....

.....

......

.....

.....

.....

......

.....

......

.....

.....

.

. . . . . . . .Review

. . . .P1

. . . . . .P2

. . . . .P3

. . . . . .P4

.Wrap-up

Solution (c) - Wald Asymptotic size α test (cont’d)

..3 Construct a two-sided asymptotic size α Wald test, whose rejectionregion is

|Z(X)| =

∣∣∣∣W(X)− θ0S/

√n

∣∣∣∣

=

∣∣∣∣∣∣n∑n

i=1 Xi− θ0

1√∑ni=1(Xi−X)2

∣∣∣∣∣∣=

∣∣∣∣ 1X − θ0

∣∣∣∣√√√√ n∑

i=1

(Xi − X)2 ≥ zα/2

Hyun Min Kang Biostatistics 602 - Lecture 26 Apil 23rd, 2013 18 / 31

Page 91: Biostatistics 602 - Statistical Inference Lecture 26 Final ... · Biostatistics 602 - Statistical Inference Lecture 26 Final Exam Review & Practice Problems for the Final Hyun Min

..........

.....

......

.....

.....

.....

......

.....

.....

.....

......

.....

.....

.....

......

.....

......

.....

.....

.

. . . . . . . .Review

. . . .P1

. . . . . .P2

. . . . .P3

. . . . . .P4

.Wrap-up

Solution (c) - Wald Asymptotic size α test (cont’d)

..3 Construct a two-sided asymptotic size α Wald test, whose rejectionregion is

|Z(X)| =

∣∣∣∣W(X)− θ0S/

√n

∣∣∣∣=

∣∣∣∣∣∣n∑n

i=1 Xi− θ0

1√∑ni=1(Xi−X)2

∣∣∣∣∣∣

=

∣∣∣∣ 1X − θ0

∣∣∣∣√√√√ n∑

i=1

(Xi − X)2 ≥ zα/2

Hyun Min Kang Biostatistics 602 - Lecture 26 Apil 23rd, 2013 18 / 31

Page 92: Biostatistics 602 - Statistical Inference Lecture 26 Final ... · Biostatistics 602 - Statistical Inference Lecture 26 Final Exam Review & Practice Problems for the Final Hyun Min

..........

.....

......

.....

.....

.....

......

.....

.....

.....

......

.....

.....

.....

......

.....

......

.....

.....

.

. . . . . . . .Review

. . . .P1

. . . . . .P2

. . . . .P3

. . . . . .P4

.Wrap-up

Solution (c) - Wald Asymptotic size α test (cont’d)

..3 Construct a two-sided asymptotic size α Wald test, whose rejectionregion is

|Z(X)| =

∣∣∣∣W(X)− θ0S/

√n

∣∣∣∣=

∣∣∣∣∣∣n∑n

i=1 Xi− θ0

1√∑ni=1(Xi−X)2

∣∣∣∣∣∣=

∣∣∣∣ 1X − θ0

∣∣∣∣√√√√ n∑

i=1

(Xi − X)2 ≥ zα/2

Hyun Min Kang Biostatistics 602 - Lecture 26 Apil 23rd, 2013 18 / 31

Page 93: Biostatistics 602 - Statistical Inference Lecture 26 Final ... · Biostatistics 602 - Statistical Inference Lecture 26 Final Exam Review & Practice Problems for the Final Hyun Min

..........

.....

......

.....

.....

.....

......

.....

.....

.....

......

.....

.....

.....

......

.....

......

.....

.....

.

. . . . . . . .Review

. . . .P1

. . . . . .P2

. . . . .P3

. . . . . .P4

.Wrap-up

Solution (d) - Asymptotic 1− α confidence intervalThe acceptance region is

A =

x :

∣∣∣∣1x − θ0

∣∣∣∣√√√√ n∑

i=1

(xi − x)2 ≤ zα/2

By inverting the acceptance region, the confidence interval is

C(X) =

θ :

∣∣∣∣ 1X − θ

∣∣∣∣√√√√ n∑

i=1

(Xi − X)2 ≤ zα/2

which is equivalent to

C(X) =

θ ∈

1

X−

zα/2√∑ni=1(Xi − X)2

,1

X+

zα/2√∑ni=1(Xi − X)2

Hyun Min Kang Biostatistics 602 - Lecture 26 Apil 23rd, 2013 19 / 31

Page 94: Biostatistics 602 - Statistical Inference Lecture 26 Final ... · Biostatistics 602 - Statistical Inference Lecture 26 Final Exam Review & Practice Problems for the Final Hyun Min

..........

.....

......

.....

.....

.....

......

.....

.....

.....

......

.....

.....

.....

......

.....

......

.....

.....

.

. . . . . . . .Review

. . . .P1

. . . . . .P2

. . . . .P3

. . . . . .P4

.Wrap-up

Solution (d) - Asymptotic 1− α confidence intervalThe acceptance region is

A =

x :

∣∣∣∣1x − θ0

∣∣∣∣√√√√ n∑

i=1

(xi − x)2 ≤ zα/2

By inverting the acceptance region, the confidence interval is

C(X) =

θ :

∣∣∣∣ 1X − θ

∣∣∣∣√√√√ n∑

i=1

(Xi − X)2 ≤ zα/2

which is equivalent to

C(X) =

θ ∈

1

X−

zα/2√∑ni=1(Xi − X)2

,1

X+

zα/2√∑ni=1(Xi − X)2

Hyun Min Kang Biostatistics 602 - Lecture 26 Apil 23rd, 2013 19 / 31

Page 95: Biostatistics 602 - Statistical Inference Lecture 26 Final ... · Biostatistics 602 - Statistical Inference Lecture 26 Final Exam Review & Practice Problems for the Final Hyun Min

..........

.....

......

.....

.....

.....

......

.....

.....

.....

......

.....

.....

.....

......

.....

......

.....

.....

.

. . . . . . . .Review

. . . .P1

. . . . . .P2

. . . . .P3

. . . . . .P4

.Wrap-up

Solution (d) - Asymptotic 1− α confidence intervalThe acceptance region is

A =

x :

∣∣∣∣1x − θ0

∣∣∣∣√√√√ n∑

i=1

(xi − x)2 ≤ zα/2

By inverting the acceptance region, the confidence interval is

C(X) =

θ :

∣∣∣∣ 1X − θ

∣∣∣∣√√√√ n∑

i=1

(Xi − X)2 ≤ zα/2

which is equivalent to

C(X) =

θ ∈

1

X−

zα/2√∑ni=1(Xi − X)2

,1

X+

zα/2√∑ni=1(Xi − X)2

Hyun Min Kang Biostatistics 602 - Lecture 26 Apil 23rd, 2013 19 / 31

Page 96: Biostatistics 602 - Statistical Inference Lecture 26 Final ... · Biostatistics 602 - Statistical Inference Lecture 26 Final Exam Review & Practice Problems for the Final Hyun Min

..........

.....

......

.....

.....

.....

......

.....

.....

.....

......

.....

.....

.....

......

.....

......

.....

.....

.

. . . . . . . .Review

. . . .P1

. . . . . .P2

. . . . .P3

. . . . . .P4

.Wrap-up

Solution (d) - Asymptotic 1− α confidence intervalThe acceptance region is

A =

x :

∣∣∣∣1x − θ0

∣∣∣∣√√√√ n∑

i=1

(xi − x)2 ≤ zα/2

By inverting the acceptance region, the confidence interval is

C(X) =

θ :

∣∣∣∣ 1X − θ

∣∣∣∣√√√√ n∑

i=1

(Xi − X)2 ≤ zα/2

which is equivalent to

C(X) =

θ ∈

1

X−

zα/2√∑ni=1(Xi − X)2

,1

X+

zα/2√∑ni=1(Xi − X)2

Hyun Min Kang Biostatistics 602 - Lecture 26 Apil 23rd, 2013 19 / 31

Page 97: Biostatistics 602 - Statistical Inference Lecture 26 Final ... · Biostatistics 602 - Statistical Inference Lecture 26 Final Exam Review & Practice Problems for the Final Hyun Min

..........

.....

......

.....

.....

.....

......

.....

.....

.....

......

.....

.....

.....

......

.....

......

.....

.....

.

. . . . . . . .Review

. . . .P1

. . . . . .P2

. . . . .P3

. . . . . .P4

.Wrap-up

Practice Problem 3

.Problem..

......

The independent random variables X1, · · · ,Xn have the following pdf

f(x|θ, β) =βxβ−1

θβ0 < x < θ, β > 0

..1 Find the MLEs of β and θ

..2 When β is a known constant β0, construct a LRT testing H0 : θ ≥ θ0vs. H1 : θ < θ0.

..3 When β is a known constant β0, find the upper confidence limit for θwith confidence coefficient 1− α.

Hyun Min Kang Biostatistics 602 - Lecture 26 Apil 23rd, 2013 20 / 31

Page 98: Biostatistics 602 - Statistical Inference Lecture 26 Final ... · Biostatistics 602 - Statistical Inference Lecture 26 Final Exam Review & Practice Problems for the Final Hyun Min

..........

.....

......

.....

.....

.....

......

.....

.....

.....

......

.....

.....

.....

......

.....

......

.....

.....

.

. . . . . . . .Review

. . . .P1

. . . . . .P2

. . . . .P3

. . . . . .P4

.Wrap-up

Practice Problem 3

.Problem..

......

The independent random variables X1, · · · ,Xn have the following pdf

f(x|θ, β) =βxβ−1

θβ0 < x < θ, β > 0

..1 Find the MLEs of β and θ

..2 When β is a known constant β0, construct a LRT testing H0 : θ ≥ θ0vs. H1 : θ < θ0.

..3 When β is a known constant β0, find the upper confidence limit for θwith confidence coefficient 1− α.

Hyun Min Kang Biostatistics 602 - Lecture 26 Apil 23rd, 2013 20 / 31

Page 99: Biostatistics 602 - Statistical Inference Lecture 26 Final ... · Biostatistics 602 - Statistical Inference Lecture 26 Final Exam Review & Practice Problems for the Final Hyun Min

..........

.....

......

.....

.....

.....

......

.....

.....

.....

......

.....

.....

.....

......

.....

......

.....

.....

.

. . . . . . . .Review

. . . .P1

. . . . . .P2

. . . . .P3

. . . . . .P4

.Wrap-up

Practice Problem 3

.Problem..

......

The independent random variables X1, · · · ,Xn have the following pdf

f(x|θ, β) =βxβ−1

θβ0 < x < θ, β > 0

..1 Find the MLEs of β and θ

..2 When β is a known constant β0, construct a LRT testing H0 : θ ≥ θ0vs. H1 : θ < θ0.

..3 When β is a known constant β0, find the upper confidence limit for θwith confidence coefficient 1− α.

Hyun Min Kang Biostatistics 602 - Lecture 26 Apil 23rd, 2013 20 / 31

Page 100: Biostatistics 602 - Statistical Inference Lecture 26 Final ... · Biostatistics 602 - Statistical Inference Lecture 26 Final Exam Review & Practice Problems for the Final Hyun Min

..........

.....

......

.....

.....

.....

......

.....

.....

.....

......

.....

.....

.....

......

.....

......

.....

.....

.

. . . . . . . .Review

. . . .P1

. . . . . .P2

. . . . .P3

. . . . . .P4

.Wrap-up

Practice Problem 3

.Problem..

......

The independent random variables X1, · · · ,Xn have the following pdf

f(x|θ, β) =βxβ−1

θβ0 < x < θ, β > 0

..1 Find the MLEs of β and θ

..2 When β is a known constant β0, construct a LRT testing H0 : θ ≥ θ0vs. H1 : θ < θ0.

..3 When β is a known constant β0, find the upper confidence limit for θwith confidence coefficient 1− α.

Hyun Min Kang Biostatistics 602 - Lecture 26 Apil 23rd, 2013 20 / 31

Page 101: Biostatistics 602 - Statistical Inference Lecture 26 Final ... · Biostatistics 602 - Statistical Inference Lecture 26 Final Exam Review & Practice Problems for the Final Hyun Min

..........

.....

......

.....

.....

.....

......

.....

.....

.....

......

.....

.....

.....

......

.....

......

.....

.....

.

. . . . . . . .Review

. . . .P1

. . . . . .P2

. . . . .P3

. . . . . .P4

.Wrap-up

(a) - MLE

L(θ, β|x) =βn (

∏ni=1 xi)

β−1

θnβ I(x(n) ≤ θ)

Because L is a decreasing function of θ and positive only when θ ≥ x(n)θ = x(n)

l(θ, β|x) = n logβ + (β − 1)∑

log xi − nβ log θ∂l∂β

=nβ+∑

log xi − n log θ = 0

β =n

n log θ −∑

log xi

=n

nx(n) −∑

log xi

Hyun Min Kang Biostatistics 602 - Lecture 26 Apil 23rd, 2013 21 / 31

Page 102: Biostatistics 602 - Statistical Inference Lecture 26 Final ... · Biostatistics 602 - Statistical Inference Lecture 26 Final Exam Review & Practice Problems for the Final Hyun Min

..........

.....

......

.....

.....

.....

......

.....

.....

.....

......

.....

.....

.....

......

.....

......

.....

.....

.

. . . . . . . .Review

. . . .P1

. . . . . .P2

. . . . .P3

. . . . . .P4

.Wrap-up

(a) - MLE

L(θ, β|x) =βn (

∏ni=1 xi)

β−1

θnβ I(x(n) ≤ θ)

Because L is a decreasing function of θ and positive only when θ ≥ x(n)

θ = x(n)l(θ, β|x) = n logβ + (β − 1)

∑log xi − nβ log θ

∂l∂β

=nβ+∑

log xi − n log θ = 0

β =n

n log θ −∑

log xi

=n

nx(n) −∑

log xi

Hyun Min Kang Biostatistics 602 - Lecture 26 Apil 23rd, 2013 21 / 31

Page 103: Biostatistics 602 - Statistical Inference Lecture 26 Final ... · Biostatistics 602 - Statistical Inference Lecture 26 Final Exam Review & Practice Problems for the Final Hyun Min

..........

.....

......

.....

.....

.....

......

.....

.....

.....

......

.....

.....

.....

......

.....

......

.....

.....

.

. . . . . . . .Review

. . . .P1

. . . . . .P2

. . . . .P3

. . . . . .P4

.Wrap-up

(a) - MLE

L(θ, β|x) =βn (

∏ni=1 xi)

β−1

θnβ I(x(n) ≤ θ)

Because L is a decreasing function of θ and positive only when θ ≥ x(n)θ = x(n)

l(θ, β|x) = n logβ + (β − 1)∑

log xi − nβ log θ∂l∂β

=nβ+∑

log xi − n log θ = 0

β =n

n log θ −∑

log xi

=n

nx(n) −∑

log xi

Hyun Min Kang Biostatistics 602 - Lecture 26 Apil 23rd, 2013 21 / 31

Page 104: Biostatistics 602 - Statistical Inference Lecture 26 Final ... · Biostatistics 602 - Statistical Inference Lecture 26 Final Exam Review & Practice Problems for the Final Hyun Min

..........

.....

......

.....

.....

.....

......

.....

.....

.....

......

.....

.....

.....

......

.....

......

.....

.....

.

. . . . . . . .Review

. . . .P1

. . . . . .P2

. . . . .P3

. . . . . .P4

.Wrap-up

(a) - MLE

L(θ, β|x) =βn (

∏ni=1 xi)

β−1

θnβ I(x(n) ≤ θ)

Because L is a decreasing function of θ and positive only when θ ≥ x(n)θ = x(n)

l(θ, β|x) = n logβ + (β − 1)∑

log xi − nβ log θ

∂l∂β

=nβ+∑

log xi − n log θ = 0

β =n

n log θ −∑

log xi

=n

nx(n) −∑

log xi

Hyun Min Kang Biostatistics 602 - Lecture 26 Apil 23rd, 2013 21 / 31

Page 105: Biostatistics 602 - Statistical Inference Lecture 26 Final ... · Biostatistics 602 - Statistical Inference Lecture 26 Final Exam Review & Practice Problems for the Final Hyun Min

..........

.....

......

.....

.....

.....

......

.....

.....

.....

......

.....

.....

.....

......

.....

......

.....

.....

.

. . . . . . . .Review

. . . .P1

. . . . . .P2

. . . . .P3

. . . . . .P4

.Wrap-up

(a) - MLE

L(θ, β|x) =βn (

∏ni=1 xi)

β−1

θnβ I(x(n) ≤ θ)

Because L is a decreasing function of θ and positive only when θ ≥ x(n)θ = x(n)

l(θ, β|x) = n logβ + (β − 1)∑

log xi − nβ log θ∂l∂β

=nβ+∑

log xi − n log θ = 0

β =n

n log θ −∑

log xi

=n

nx(n) −∑

log xi

Hyun Min Kang Biostatistics 602 - Lecture 26 Apil 23rd, 2013 21 / 31

Page 106: Biostatistics 602 - Statistical Inference Lecture 26 Final ... · Biostatistics 602 - Statistical Inference Lecture 26 Final Exam Review & Practice Problems for the Final Hyun Min

..........

.....

......

.....

.....

.....

......

.....

.....

.....

......

.....

.....

.....

......

.....

......

.....

.....

.

. . . . . . . .Review

. . . .P1

. . . . . .P2

. . . . .P3

. . . . . .P4

.Wrap-up

(a) - MLE

L(θ, β|x) =βn (

∏ni=1 xi)

β−1

θnβ I(x(n) ≤ θ)

Because L is a decreasing function of θ and positive only when θ ≥ x(n)θ = x(n)

l(θ, β|x) = n logβ + (β − 1)∑

log xi − nβ log θ∂l∂β

=nβ+∑

log xi − n log θ = 0

β =n

n log θ −∑

log xi

=n

nx(n) −∑

log xi

Hyun Min Kang Biostatistics 602 - Lecture 26 Apil 23rd, 2013 21 / 31

Page 107: Biostatistics 602 - Statistical Inference Lecture 26 Final ... · Biostatistics 602 - Statistical Inference Lecture 26 Final Exam Review & Practice Problems for the Final Hyun Min

..........

.....

......

.....

.....

.....

......

.....

.....

.....

......

.....

.....

.....

......

.....

......

.....

.....

.

. . . . . . . .Review

. . . .P1

. . . . . .P2

. . . . .P3

. . . . . .P4

.Wrap-up

(a) - MLE

L(θ, β|x) =βn (

∏ni=1 xi)

β−1

θnβ I(x(n) ≤ θ)

Because L is a decreasing function of θ and positive only when θ ≥ x(n)θ = x(n)

l(θ, β|x) = n logβ + (β − 1)∑

log xi − nβ log θ∂l∂β

=nβ+∑

log xi − n log θ = 0

β =n

n log θ −∑

log xi

=n

nx(n) −∑

log xi

Hyun Min Kang Biostatistics 602 - Lecture 26 Apil 23rd, 2013 21 / 31

Page 108: Biostatistics 602 - Statistical Inference Lecture 26 Final ... · Biostatistics 602 - Statistical Inference Lecture 26 Final Exam Review & Practice Problems for the Final Hyun Min

..........

.....

......

.....

.....

.....

......

.....

.....

.....

......

.....

.....

.....

......

.....

......

.....

.....

.

. . . . . . . .Review

. . . .P1

. . . . . .P2

. . . . .P3

. . . . . .P4

.Wrap-up

(b) - LRT

λ(x) =supθ∈Ω0

L(θ|x)supθ∈Ω L(θ|x)

=

1 θ0 < x(n)L(θ0|x)

L(x(n)|x) θ0 ≥ x(n)

=

1 θ0 < x(n)(x(n))

nβ0

θnβ00

θ0 ≥ x(n)≤ c

x(n)θ0

≤ c∗

Hyun Min Kang Biostatistics 602 - Lecture 26 Apil 23rd, 2013 22 / 31

Page 109: Biostatistics 602 - Statistical Inference Lecture 26 Final ... · Biostatistics 602 - Statistical Inference Lecture 26 Final Exam Review & Practice Problems for the Final Hyun Min

..........

.....

......

.....

.....

.....

......

.....

.....

.....

......

.....

.....

.....

......

.....

......

.....

.....

.

. . . . . . . .Review

. . . .P1

. . . . . .P2

. . . . .P3

. . . . . .P4

.Wrap-up

(b) - LRT

λ(x) =supθ∈Ω0

L(θ|x)supθ∈Ω L(θ|x)

=

1 θ0 < x(n)L(θ0|x)

L(x(n)|x) θ0 ≥ x(n)

=

1 θ0 < x(n)(x(n))

nβ0

θnβ00

θ0 ≥ x(n)≤ c

x(n)θ0

≤ c∗

Hyun Min Kang Biostatistics 602 - Lecture 26 Apil 23rd, 2013 22 / 31

Page 110: Biostatistics 602 - Statistical Inference Lecture 26 Final ... · Biostatistics 602 - Statistical Inference Lecture 26 Final Exam Review & Practice Problems for the Final Hyun Min

..........

.....

......

.....

.....

.....

......

.....

.....

.....

......

.....

.....

.....

......

.....

......

.....

.....

.

. . . . . . . .Review

. . . .P1

. . . . . .P2

. . . . .P3

. . . . . .P4

.Wrap-up

(b) - LRT

λ(x) =supθ∈Ω0

L(θ|x)supθ∈Ω L(θ|x)

=

1 θ0 < x(n)L(θ0|x)

L(x(n)|x) θ0 ≥ x(n)

=

1 θ0 < x(n)(x(n))

nβ0

θnβ00

θ0 ≥ x(n)≤ c

x(n)θ0

≤ c∗

Hyun Min Kang Biostatistics 602 - Lecture 26 Apil 23rd, 2013 22 / 31

Page 111: Biostatistics 602 - Statistical Inference Lecture 26 Final ... · Biostatistics 602 - Statistical Inference Lecture 26 Final Exam Review & Practice Problems for the Final Hyun Min

..........

.....

......

.....

.....

.....

......

.....

.....

.....

......

.....

.....

.....

......

.....

......

.....

.....

.

. . . . . . . .Review

. . . .P1

. . . . . .P2

. . . . .P3

. . . . . .P4

.Wrap-up

(b) - LRT

λ(x) =supθ∈Ω0

L(θ|x)supθ∈Ω L(θ|x)

=

1 θ0 < x(n)L(θ0|x)

L(x(n)|x) θ0 ≥ x(n)

=

1 θ0 < x(n)(x(n))

nβ0

θnβ00

θ0 ≥ x(n)≤ c

x(n)θ0

≤ c∗

Hyun Min Kang Biostatistics 602 - Lecture 26 Apil 23rd, 2013 22 / 31

Page 112: Biostatistics 602 - Statistical Inference Lecture 26 Final ... · Biostatistics 602 - Statistical Inference Lecture 26 Final Exam Review & Practice Problems for the Final Hyun Min

..........

.....

......

.....

.....

.....

......

.....

.....

.....

......

.....

.....

.....

......

.....

......

.....

.....

.

. . . . . . . .Review

. . . .P1

. . . . . .P2

. . . . .P3

. . . . . .P4

.Wrap-up

(b) - size α LRT

α = Pr(x(n)

θ0≤ c∗

)

= (c∗)nβ0

c∗ = α1

nβ0

Therefore, the rejection region for size α LRT is is

R =

x : x(n) ≤ θ0α1

nβ0

Hyun Min Kang Biostatistics 602 - Lecture 26 Apil 23rd, 2013 23 / 31

Page 113: Biostatistics 602 - Statistical Inference Lecture 26 Final ... · Biostatistics 602 - Statistical Inference Lecture 26 Final Exam Review & Practice Problems for the Final Hyun Min

..........

.....

......

.....

.....

.....

......

.....

.....

.....

......

.....

.....

.....

......

.....

......

.....

.....

.

. . . . . . . .Review

. . . .P1

. . . . . .P2

. . . . .P3

. . . . . .P4

.Wrap-up

(b) - size α LRT

α = Pr(x(n)

θ0≤ c∗

)= (c∗)nβ0

c∗ = α1

nβ0

Therefore, the rejection region for size α LRT is is

R =

x : x(n) ≤ θ0α1

nβ0

Hyun Min Kang Biostatistics 602 - Lecture 26 Apil 23rd, 2013 23 / 31

Page 114: Biostatistics 602 - Statistical Inference Lecture 26 Final ... · Biostatistics 602 - Statistical Inference Lecture 26 Final Exam Review & Practice Problems for the Final Hyun Min

..........

.....

......

.....

.....

.....

......

.....

.....

.....

......

.....

.....

.....

......

.....

......

.....

.....

.

. . . . . . . .Review

. . . .P1

. . . . . .P2

. . . . .P3

. . . . . .P4

.Wrap-up

(b) - size α LRT

α = Pr(x(n)

θ0≤ c∗

)= (c∗)nβ0

c∗ = α1

nβ0

Therefore, the rejection region for size α LRT is is

R =

x : x(n) ≤ θ0α1

nβ0

Hyun Min Kang Biostatistics 602 - Lecture 26 Apil 23rd, 2013 23 / 31

Page 115: Biostatistics 602 - Statistical Inference Lecture 26 Final ... · Biostatistics 602 - Statistical Inference Lecture 26 Final Exam Review & Practice Problems for the Final Hyun Min

..........

.....

......

.....

.....

.....

......

.....

.....

.....

......

.....

.....

.....

......

.....

......

.....

.....

.

. . . . . . . .Review

. . . .P1

. . . . . .P2

. . . . .P3

. . . . . .P4

.Wrap-up

(b) - size α LRT

α = Pr(x(n)

θ0≤ c∗

)= (c∗)nβ0

c∗ = α1

nβ0

Therefore, the rejection region for size α LRT is is

R =

x : x(n) ≤ θ0α1

nβ0

Hyun Min Kang Biostatistics 602 - Lecture 26 Apil 23rd, 2013 23 / 31

Page 116: Biostatistics 602 - Statistical Inference Lecture 26 Final ... · Biostatistics 602 - Statistical Inference Lecture 26 Final Exam Review & Practice Problems for the Final Hyun Min

..........

.....

......

.....

.....

.....

......

.....

.....

.....

......

.....

.....

.....

......

.....

......

.....

.....

.

. . . . . . . .Review

. . . .P1

. . . . . .P2

. . . . .P3

. . . . . .P4

.Wrap-up

(b) - size α LRT

α = Pr(x(n)

θ0≤ c∗

)= (c∗)nβ0

c∗ = α1

nβ0

Therefore, the rejection region for size α LRT is is

R =

x : x(n) ≤ θ0α1

nβ0

Hyun Min Kang Biostatistics 602 - Lecture 26 Apil 23rd, 2013 23 / 31

Page 117: Biostatistics 602 - Statistical Inference Lecture 26 Final ... · Biostatistics 602 - Statistical Inference Lecture 26 Final Exam Review & Practice Problems for the Final Hyun Min

..........

.....

......

.....

.....

.....

......

.....

.....

.....

......

.....

.....

.....

......

.....

......

.....

.....

.

. . . . . . . .Review

. . . .P1

. . . . . .P2

. . . . .P3

. . . . . .P4

.Wrap-up

(c) - Upper 1− α confidence limit

The acceptance region of size α LRT is

A(θ0) =

x : x(n) > θ0α1

nβ0

By inserting the acceptance region, the 1− α confidence interval becomes

C(X) =θ : X(n) > θα

1nβ0

=

θ : θ < X(n)α

− 1nβ0

Therefore, the upper 1− α confidence limit is X(n)α− 1

nβ0 .

Hyun Min Kang Biostatistics 602 - Lecture 26 Apil 23rd, 2013 24 / 31

Page 118: Biostatistics 602 - Statistical Inference Lecture 26 Final ... · Biostatistics 602 - Statistical Inference Lecture 26 Final Exam Review & Practice Problems for the Final Hyun Min

..........

.....

......

.....

.....

.....

......

.....

.....

.....

......

.....

.....

.....

......

.....

......

.....

.....

.

. . . . . . . .Review

. . . .P1

. . . . . .P2

. . . . .P3

. . . . . .P4

.Wrap-up

(c) - Upper 1− α confidence limit

The acceptance region of size α LRT is

A(θ0) =

x : x(n) > θ0α1

nβ0

By inserting the acceptance region, the 1− α confidence interval becomes

C(X) =θ : X(n) > θα

1nβ0

=

θ : θ < X(n)α

− 1nβ0

Therefore, the upper 1− α confidence limit is X(n)α− 1

nβ0 .

Hyun Min Kang Biostatistics 602 - Lecture 26 Apil 23rd, 2013 24 / 31

Page 119: Biostatistics 602 - Statistical Inference Lecture 26 Final ... · Biostatistics 602 - Statistical Inference Lecture 26 Final Exam Review & Practice Problems for the Final Hyun Min

..........

.....

......

.....

.....

.....

......

.....

.....

.....

......

.....

.....

.....

......

.....

......

.....

.....

.

. . . . . . . .Review

. . . .P1

. . . . . .P2

. . . . .P3

. . . . . .P4

.Wrap-up

(c) - Upper 1− α confidence limit

The acceptance region of size α LRT is

A(θ0) =

x : x(n) > θ0α1

nβ0

By inserting the acceptance region, the 1− α confidence interval becomes

C(X) =θ : X(n) > θα

1nβ0

=

θ : θ < X(n)α

− 1nβ0

Therefore, the upper 1− α confidence limit is X(n)α− 1

nβ0 .

Hyun Min Kang Biostatistics 602 - Lecture 26 Apil 23rd, 2013 24 / 31

Page 120: Biostatistics 602 - Statistical Inference Lecture 26 Final ... · Biostatistics 602 - Statistical Inference Lecture 26 Final Exam Review & Practice Problems for the Final Hyun Min

..........

.....

......

.....

.....

.....

......

.....

.....

.....

......

.....

.....

.....

......

.....

......

.....

.....

.

. . . . . . . .Review

. . . .P1

. . . . . .P2

. . . . .P3

. . . . . .P4

.Wrap-up

(c) - Upper 1− α confidence limit

The acceptance region of size α LRT is

A(θ0) =

x : x(n) > θ0α1

nβ0

By inserting the acceptance region, the 1− α confidence interval becomes

C(X) =θ : X(n) > θα

1nβ0

=θ : θ < X(n)α

− 1nβ0

Therefore, the upper 1− α confidence limit is X(n)α− 1

nβ0 .

Hyun Min Kang Biostatistics 602 - Lecture 26 Apil 23rd, 2013 24 / 31

Page 121: Biostatistics 602 - Statistical Inference Lecture 26 Final ... · Biostatistics 602 - Statistical Inference Lecture 26 Final Exam Review & Practice Problems for the Final Hyun Min

..........

.....

......

.....

.....

.....

......

.....

.....

.....

......

.....

.....

.....

......

.....

......

.....

.....

.

. . . . . . . .Review

. . . .P1

. . . . . .P2

. . . . .P3

. . . . . .P4

.Wrap-up

(c) - Upper 1− α confidence limit

The acceptance region of size α LRT is

A(θ0) =

x : x(n) > θ0α1

nβ0

By inserting the acceptance region, the 1− α confidence interval becomes

C(X) =θ : X(n) > θα

1nβ0

=

θ : θ < X(n)α

− 1nβ0

Therefore, the upper 1− α confidence limit is X(n)α− 1

nβ0 .

Hyun Min Kang Biostatistics 602 - Lecture 26 Apil 23rd, 2013 24 / 31

Page 122: Biostatistics 602 - Statistical Inference Lecture 26 Final ... · Biostatistics 602 - Statistical Inference Lecture 26 Final Exam Review & Practice Problems for the Final Hyun Min

..........

.....

......

.....

.....

.....

......

.....

.....

.....

......

.....

.....

.....

......

.....

......

.....

.....

.

. . . . . . . .Review

. . . .P1

. . . . . .P2

. . . . .P3

. . . . . .P4

.Wrap-up

(c) - Upper 1− α confidence limit

The acceptance region of size α LRT is

A(θ0) =

x : x(n) > θ0α1

nβ0

By inserting the acceptance region, the 1− α confidence interval becomes

C(X) =θ : X(n) > θα

1nβ0

=

θ : θ < X(n)α

− 1nβ0

Therefore, the upper 1− α confidence limit is X(n)α− 1

nβ0 .

Hyun Min Kang Biostatistics 602 - Lecture 26 Apil 23rd, 2013 24 / 31

Page 123: Biostatistics 602 - Statistical Inference Lecture 26 Final ... · Biostatistics 602 - Statistical Inference Lecture 26 Final Exam Review & Practice Problems for the Final Hyun Min

..........

.....

......

.....

.....

.....

......

.....

.....

.....

......

.....

.....

.....

......

.....

......

.....

.....

.

. . . . . . . .Review

. . . .P1

. . . . . .P2

. . . . .P3

. . . . . .P4

.Wrap-up

Practice Problem 4

.Problem..

......

A random sample X1, · · · ,Xn is drawn from a population N (θ, θ) whereθ > 0.

(a) Find the θ, the MLE of θ(b) Find the asymptotic distribution of θ.(c) Compute ARE(θ,X). Determine whether θ is asymptotically more

efficient than X or not.You may use the following fact: Var(X2) = 4θ3 + 2θ2.

Hyun Min Kang Biostatistics 602 - Lecture 26 Apil 23rd, 2013 25 / 31

Page 124: Biostatistics 602 - Statistical Inference Lecture 26 Final ... · Biostatistics 602 - Statistical Inference Lecture 26 Final Exam Review & Practice Problems for the Final Hyun Min

..........

.....

......

.....

.....

.....

......

.....

.....

.....

......

.....

.....

.....

......

.....

......

.....

.....

.

. . . . . . . .Review

. . . .P1

. . . . . .P2

. . . . .P3

. . . . . .P4

.Wrap-up

Practice Problem 4

.Problem..

......

A random sample X1, · · · ,Xn is drawn from a population N (θ, θ) whereθ > 0.(a) Find the θ, the MLE of θ

(b) Find the asymptotic distribution of θ.(c) Compute ARE(θ,X). Determine whether θ is asymptotically more

efficient than X or not.You may use the following fact: Var(X2) = 4θ3 + 2θ2.

Hyun Min Kang Biostatistics 602 - Lecture 26 Apil 23rd, 2013 25 / 31

Page 125: Biostatistics 602 - Statistical Inference Lecture 26 Final ... · Biostatistics 602 - Statistical Inference Lecture 26 Final Exam Review & Practice Problems for the Final Hyun Min

..........

.....

......

.....

.....

.....

......

.....

.....

.....

......

.....

.....

.....

......

.....

......

.....

.....

.

. . . . . . . .Review

. . . .P1

. . . . . .P2

. . . . .P3

. . . . . .P4

.Wrap-up

Practice Problem 4

.Problem..

......

A random sample X1, · · · ,Xn is drawn from a population N (θ, θ) whereθ > 0.(a) Find the θ, the MLE of θ(b) Find the asymptotic distribution of θ.

(c) Compute ARE(θ,X). Determine whether θ is asymptotically moreefficient than X or not.

You may use the following fact: Var(X2) = 4θ3 + 2θ2.

Hyun Min Kang Biostatistics 602 - Lecture 26 Apil 23rd, 2013 25 / 31

Page 126: Biostatistics 602 - Statistical Inference Lecture 26 Final ... · Biostatistics 602 - Statistical Inference Lecture 26 Final Exam Review & Practice Problems for the Final Hyun Min

..........

.....

......

.....

.....

.....

......

.....

.....

.....

......

.....

.....

.....

......

.....

......

.....

.....

.

. . . . . . . .Review

. . . .P1

. . . . . .P2

. . . . .P3

. . . . . .P4

.Wrap-up

Practice Problem 4

.Problem..

......

A random sample X1, · · · ,Xn is drawn from a population N (θ, θ) whereθ > 0.(a) Find the θ, the MLE of θ(b) Find the asymptotic distribution of θ.(c) Compute ARE(θ,X). Determine whether θ is asymptotically more

efficient than X or not.

You may use the following fact: Var(X2) = 4θ3 + 2θ2.

Hyun Min Kang Biostatistics 602 - Lecture 26 Apil 23rd, 2013 25 / 31

Page 127: Biostatistics 602 - Statistical Inference Lecture 26 Final ... · Biostatistics 602 - Statistical Inference Lecture 26 Final Exam Review & Practice Problems for the Final Hyun Min

..........

.....

......

.....

.....

.....

......

.....

.....

.....

......

.....

.....

.....

......

.....

......

.....

.....

.

. . . . . . . .Review

. . . .P1

. . . . . .P2

. . . . .P3

. . . . . .P4

.Wrap-up

Practice Problem 4

.Problem..

......

A random sample X1, · · · ,Xn is drawn from a population N (θ, θ) whereθ > 0.(a) Find the θ, the MLE of θ(b) Find the asymptotic distribution of θ.(c) Compute ARE(θ,X). Determine whether θ is asymptotically more

efficient than X or not.You may use the following fact: Var(X2) = 4θ3 + 2θ2.

Hyun Min Kang Biostatistics 602 - Lecture 26 Apil 23rd, 2013 25 / 31

Page 128: Biostatistics 602 - Statistical Inference Lecture 26 Final ... · Biostatistics 602 - Statistical Inference Lecture 26 Final Exam Review & Practice Problems for the Final Hyun Min

..........

.....

......

.....

.....

.....

......

.....

.....

.....

......

.....

.....

.....

......

.....

......

.....

.....

.

. . . . . . . .Review

. . . .P1

. . . . . .P2

. . . . .P3

. . . . . .P4

.Wrap-up

(a) - MLE of θ

L(θ|x) = (2πθ)n/2 exp[−∑n

i=1(xi − θ)2

]

l(θ|x) =n2

log(2π) + n2

log θ −∑n

i=1(xi − θ)2

=n2

log(2π) + n2

log θ −∑

x2i2θ

+∑

xi −nθ2

l′(θ|x) =n2θ

+

∑x2i

2θ2− n

2=

nθ −∑

x2i − nθ22θ2

= 0

nθ2 + nθ −∑

x2i = 0

θ =−1 +

√1 + 4

∑x2i /n

21

n∑

x2i = θ2 + θ

Hyun Min Kang Biostatistics 602 - Lecture 26 Apil 23rd, 2013 26 / 31

Page 129: Biostatistics 602 - Statistical Inference Lecture 26 Final ... · Biostatistics 602 - Statistical Inference Lecture 26 Final Exam Review & Practice Problems for the Final Hyun Min

..........

.....

......

.....

.....

.....

......

.....

.....

.....

......

.....

.....

.....

......

.....

......

.....

.....

.

. . . . . . . .Review

. . . .P1

. . . . . .P2

. . . . .P3

. . . . . .P4

.Wrap-up

(a) - MLE of θ

L(θ|x) = (2πθ)n/2 exp[−∑n

i=1(xi − θ)2

]l(θ|x) =

n2

log(2π) + n2

log θ −∑n

i=1(xi − θ)2

=n2

log(2π) + n2

log θ −∑

x2i2θ

+∑

xi −nθ2

l′(θ|x) =n2θ

+

∑x2i

2θ2− n

2=

nθ −∑

x2i − nθ22θ2

= 0

nθ2 + nθ −∑

x2i = 0

θ =−1 +

√1 + 4

∑x2i /n

21

n∑

x2i = θ2 + θ

Hyun Min Kang Biostatistics 602 - Lecture 26 Apil 23rd, 2013 26 / 31

Page 130: Biostatistics 602 - Statistical Inference Lecture 26 Final ... · Biostatistics 602 - Statistical Inference Lecture 26 Final Exam Review & Practice Problems for the Final Hyun Min

..........

.....

......

.....

.....

.....

......

.....

.....

.....

......

.....

.....

.....

......

.....

......

.....

.....

.

. . . . . . . .Review

. . . .P1

. . . . . .P2

. . . . .P3

. . . . . .P4

.Wrap-up

(a) - MLE of θ

L(θ|x) = (2πθ)n/2 exp[−∑n

i=1(xi − θ)2

]l(θ|x) =

n2

log(2π) + n2

log θ −∑n

i=1(xi − θ)2

=n2

log(2π) + n2

log θ −∑

x2i2θ

+∑

xi −nθ2

l′(θ|x) =n2θ

+

∑x2i

2θ2− n

2=

nθ −∑

x2i − nθ22θ2

= 0

nθ2 + nθ −∑

x2i = 0

θ =−1 +

√1 + 4

∑x2i /n

21

n∑

x2i = θ2 + θ

Hyun Min Kang Biostatistics 602 - Lecture 26 Apil 23rd, 2013 26 / 31

Page 131: Biostatistics 602 - Statistical Inference Lecture 26 Final ... · Biostatistics 602 - Statistical Inference Lecture 26 Final Exam Review & Practice Problems for the Final Hyun Min

..........

.....

......

.....

.....

.....

......

.....

.....

.....

......

.....

.....

.....

......

.....

......

.....

.....

.

. . . . . . . .Review

. . . .P1

. . . . . .P2

. . . . .P3

. . . . . .P4

.Wrap-up

(a) - MLE of θ

L(θ|x) = (2πθ)n/2 exp[−∑n

i=1(xi − θ)2

]l(θ|x) =

n2

log(2π) + n2

log θ −∑n

i=1(xi − θ)2

=n2

log(2π) + n2

log θ −∑

x2i2θ

+∑

xi −nθ2

l′(θ|x) =n2θ

+

∑x2i

2θ2− n

2=

nθ −∑

x2i − nθ22θ2

= 0

nθ2 + nθ −∑

x2i = 0

θ =−1 +

√1 + 4

∑x2i /n

21

n∑

x2i = θ2 + θ

Hyun Min Kang Biostatistics 602 - Lecture 26 Apil 23rd, 2013 26 / 31

Page 132: Biostatistics 602 - Statistical Inference Lecture 26 Final ... · Biostatistics 602 - Statistical Inference Lecture 26 Final Exam Review & Practice Problems for the Final Hyun Min

..........

.....

......

.....

.....

.....

......

.....

.....

.....

......

.....

.....

.....

......

.....

......

.....

.....

.

. . . . . . . .Review

. . . .P1

. . . . . .P2

. . . . .P3

. . . . . .P4

.Wrap-up

(a) - MLE of θ

L(θ|x) = (2πθ)n/2 exp[−∑n

i=1(xi − θ)2

]l(θ|x) =

n2

log(2π) + n2

log θ −∑n

i=1(xi − θ)2

=n2

log(2π) + n2

log θ −∑

x2i2θ

+∑

xi −nθ2

l′(θ|x) =n2θ

+

∑x2i

2θ2− n

2=

nθ −∑

x2i − nθ22θ2

= 0

nθ2 + nθ −∑

x2i = 0

θ =−1 +

√1 + 4

∑x2i /n

21

n∑

x2i = θ2 + θ

Hyun Min Kang Biostatistics 602 - Lecture 26 Apil 23rd, 2013 26 / 31

Page 133: Biostatistics 602 - Statistical Inference Lecture 26 Final ... · Biostatistics 602 - Statistical Inference Lecture 26 Final Exam Review & Practice Problems for the Final Hyun Min

..........

.....

......

.....

.....

.....

......

.....

.....

.....

......

.....

.....

.....

......

.....

......

.....

.....

.

. . . . . . . .Review

. . . .P1

. . . . . .P2

. . . . .P3

. . . . . .P4

.Wrap-up

(a) - MLE of θ

L(θ|x) = (2πθ)n/2 exp[−∑n

i=1(xi − θ)2

]l(θ|x) =

n2

log(2π) + n2

log θ −∑n

i=1(xi − θ)2

=n2

log(2π) + n2

log θ −∑

x2i2θ

+∑

xi −nθ2

l′(θ|x) =n2θ

+

∑x2i

2θ2− n

2=

nθ −∑

x2i − nθ22θ2

= 0

nθ2 + nθ −∑

x2i = 0

θ =−1 +

√1 + 4

∑x2i /n

2

1

n∑

x2i = θ2 + θ

Hyun Min Kang Biostatistics 602 - Lecture 26 Apil 23rd, 2013 26 / 31

Page 134: Biostatistics 602 - Statistical Inference Lecture 26 Final ... · Biostatistics 602 - Statistical Inference Lecture 26 Final Exam Review & Practice Problems for the Final Hyun Min

..........

.....

......

.....

.....

.....

......

.....

.....

.....

......

.....

.....

.....

......

.....

......

.....

.....

.

. . . . . . . .Review

. . . .P1

. . . . . .P2

. . . . .P3

. . . . . .P4

.Wrap-up

(a) - MLE of θ

L(θ|x) = (2πθ)n/2 exp[−∑n

i=1(xi − θ)2

]l(θ|x) =

n2

log(2π) + n2

log θ −∑n

i=1(xi − θ)2

=n2

log(2π) + n2

log θ −∑

x2i2θ

+∑

xi −nθ2

l′(θ|x) =n2θ

+

∑x2i

2θ2− n

2=

nθ −∑

x2i − nθ22θ2

= 0

nθ2 + nθ −∑

x2i = 0

θ =−1 +

√1 + 4

∑x2i /n

21

n∑

x2i = θ2 + θ

Hyun Min Kang Biostatistics 602 - Lecture 26 Apil 23rd, 2013 26 / 31

Page 135: Biostatistics 602 - Statistical Inference Lecture 26 Final ... · Biostatistics 602 - Statistical Inference Lecture 26 Final Exam Review & Practice Problems for the Final Hyun Min

..........

.....

......

.....

.....

.....

......

.....

.....

.....

......

.....

.....

.....

......

.....

......

.....

.....

.

. . . . . . . .Review

. . . .P1

. . . . . .P2

. . . . .P3

. . . . . .P4

.Wrap-up

(b) - Asymptotic distribution of MLE

By CLT, Let W = 1n∑

X2i , then

W ∼ AN(

EX2,Var(X2)

n

)

= AN(θ + θ2,

4θ3 + 2θ2

n

)The asymptotic distribution of MLE θ

θ ∼ AN(θ,

σ2(θ)

n

)for some function σ2(θ) and we would like to find σ2(θ) using theasymptotic distribution of W.

Hyun Min Kang Biostatistics 602 - Lecture 26 Apil 23rd, 2013 27 / 31

Page 136: Biostatistics 602 - Statistical Inference Lecture 26 Final ... · Biostatistics 602 - Statistical Inference Lecture 26 Final Exam Review & Practice Problems for the Final Hyun Min

..........

.....

......

.....

.....

.....

......

.....

.....

.....

......

.....

.....

.....

......

.....

......

.....

.....

.

. . . . . . . .Review

. . . .P1

. . . . . .P2

. . . . .P3

. . . . . .P4

.Wrap-up

(b) - Asymptotic distribution of MLE

By CLT, Let W = 1n∑

X2i , then

W ∼ AN(

EX2,Var(X2)

n

)= AN

(θ + θ2,

4θ3 + 2θ2

n

)

The asymptotic distribution of MLE θ

θ ∼ AN(θ,

σ2(θ)

n

)for some function σ2(θ) and we would like to find σ2(θ) using theasymptotic distribution of W.

Hyun Min Kang Biostatistics 602 - Lecture 26 Apil 23rd, 2013 27 / 31

Page 137: Biostatistics 602 - Statistical Inference Lecture 26 Final ... · Biostatistics 602 - Statistical Inference Lecture 26 Final Exam Review & Practice Problems for the Final Hyun Min

..........

.....

......

.....

.....

.....

......

.....

.....

.....

......

.....

.....

.....

......

.....

......

.....

.....

.

. . . . . . . .Review

. . . .P1

. . . . . .P2

. . . . .P3

. . . . . .P4

.Wrap-up

(b) - Asymptotic distribution of MLE

By CLT, Let W = 1n∑

X2i , then

W ∼ AN(

EX2,Var(X2)

n

)= AN

(θ + θ2,

4θ3 + 2θ2

n

)The asymptotic distribution of MLE θ

θ ∼ AN(θ,

σ2(θ)

n

)

for some function σ2(θ) and we would like to find σ2(θ) using theasymptotic distribution of W.

Hyun Min Kang Biostatistics 602 - Lecture 26 Apil 23rd, 2013 27 / 31

Page 138: Biostatistics 602 - Statistical Inference Lecture 26 Final ... · Biostatistics 602 - Statistical Inference Lecture 26 Final Exam Review & Practice Problems for the Final Hyun Min

..........

.....

......

.....

.....

.....

......

.....

.....

.....

......

.....

.....

.....

......

.....

......

.....

.....

.

. . . . . . . .Review

. . . .P1

. . . . . .P2

. . . . .P3

. . . . . .P4

.Wrap-up

(b) - Asymptotic distribution of MLE

By CLT, Let W = 1n∑

X2i , then

W ∼ AN(

EX2,Var(X2)

n

)= AN

(θ + θ2,

4θ3 + 2θ2

n

)The asymptotic distribution of MLE θ

θ ∼ AN(θ,

σ2(θ)

n

)for some function σ2(θ) and we would like to find σ2(θ) using theasymptotic distribution of W.

Hyun Min Kang Biostatistics 602 - Lecture 26 Apil 23rd, 2013 27 / 31

Page 139: Biostatistics 602 - Statistical Inference Lecture 26 Final ... · Biostatistics 602 - Statistical Inference Lecture 26 Final Exam Review & Practice Problems for the Final Hyun Min

..........

.....

......

.....

.....

.....

......

.....

.....

.....

......

.....

.....

.....

......

.....

......

.....

.....

.

. . . . . . . .Review

. . . .P1

. . . . . .P2

. . . . .P3

. . . . . .P4

.Wrap-up

(b) - Asymptotic distribution of MLE (cont’d)

Let g(y) = y2 + y, then g′(y) = (2y + 1) and g(θ) = W. Then by theDelta Method, the asymptotic distribution of W can be written as

W = g(θ) ∼ AN(

g(θ), g′(θ)σ2(θ)

n

)= AN

(θ2 + θ,

(2θ + 1)2σ2(θ)

n

)= AN

(θ2 + θ,

4θ3 + 2θ2

n

)σ2(θ) =

4θ3 + 2θ2

(2θ + 1)2=

2θ2(2θ + 1)

(2θ + 1)2=

2θ2

2θ + 1

Hyun Min Kang Biostatistics 602 - Lecture 26 Apil 23rd, 2013 28 / 31

Page 140: Biostatistics 602 - Statistical Inference Lecture 26 Final ... · Biostatistics 602 - Statistical Inference Lecture 26 Final Exam Review & Practice Problems for the Final Hyun Min

..........

.....

......

.....

.....

.....

......

.....

.....

.....

......

.....

.....

.....

......

.....

......

.....

.....

.

. . . . . . . .Review

. . . .P1

. . . . . .P2

. . . . .P3

. . . . . .P4

.Wrap-up

(b) - Asymptotic distribution of MLE (cont’d)

Let g(y) = y2 + y, then g′(y) = (2y + 1) and g(θ) = W. Then by theDelta Method, the asymptotic distribution of W can be written as

W = g(θ) ∼ AN(

g(θ), g′(θ)σ2(θ)

n

)

= AN(θ2 + θ,

(2θ + 1)2σ2(θ)

n

)= AN

(θ2 + θ,

4θ3 + 2θ2

n

)σ2(θ) =

4θ3 + 2θ2

(2θ + 1)2=

2θ2(2θ + 1)

(2θ + 1)2=

2θ2

2θ + 1

Hyun Min Kang Biostatistics 602 - Lecture 26 Apil 23rd, 2013 28 / 31

Page 141: Biostatistics 602 - Statistical Inference Lecture 26 Final ... · Biostatistics 602 - Statistical Inference Lecture 26 Final Exam Review & Practice Problems for the Final Hyun Min

..........

.....

......

.....

.....

.....

......

.....

.....

.....

......

.....

.....

.....

......

.....

......

.....

.....

.

. . . . . . . .Review

. . . .P1

. . . . . .P2

. . . . .P3

. . . . . .P4

.Wrap-up

(b) - Asymptotic distribution of MLE (cont’d)

Let g(y) = y2 + y, then g′(y) = (2y + 1) and g(θ) = W. Then by theDelta Method, the asymptotic distribution of W can be written as

W = g(θ) ∼ AN(

g(θ), g′(θ)σ2(θ)

n

)= AN

(θ2 + θ,

(2θ + 1)2σ2(θ)

n

)

= AN(θ2 + θ,

4θ3 + 2θ2

n

)σ2(θ) =

4θ3 + 2θ2

(2θ + 1)2=

2θ2(2θ + 1)

(2θ + 1)2=

2θ2

2θ + 1

Hyun Min Kang Biostatistics 602 - Lecture 26 Apil 23rd, 2013 28 / 31

Page 142: Biostatistics 602 - Statistical Inference Lecture 26 Final ... · Biostatistics 602 - Statistical Inference Lecture 26 Final Exam Review & Practice Problems for the Final Hyun Min

..........

.....

......

.....

.....

.....

......

.....

.....

.....

......

.....

.....

.....

......

.....

......

.....

.....

.

. . . . . . . .Review

. . . .P1

. . . . . .P2

. . . . .P3

. . . . . .P4

.Wrap-up

(b) - Asymptotic distribution of MLE (cont’d)

Let g(y) = y2 + y, then g′(y) = (2y + 1) and g(θ) = W. Then by theDelta Method, the asymptotic distribution of W can be written as

W = g(θ) ∼ AN(

g(θ), g′(θ)σ2(θ)

n

)= AN

(θ2 + θ,

(2θ + 1)2σ2(θ)

n

)= AN

(θ2 + θ,

4θ3 + 2θ2

n

)

σ2(θ) =4θ3 + 2θ2

(2θ + 1)2=

2θ2(2θ + 1)

(2θ + 1)2=

2θ2

2θ + 1

Hyun Min Kang Biostatistics 602 - Lecture 26 Apil 23rd, 2013 28 / 31

Page 143: Biostatistics 602 - Statistical Inference Lecture 26 Final ... · Biostatistics 602 - Statistical Inference Lecture 26 Final Exam Review & Practice Problems for the Final Hyun Min

..........

.....

......

.....

.....

.....

......

.....

.....

.....

......

.....

.....

.....

......

.....

......

.....

.....

.

. . . . . . . .Review

. . . .P1

. . . . . .P2

. . . . .P3

. . . . . .P4

.Wrap-up

(b) - Asymptotic distribution of MLE (cont’d)

Let g(y) = y2 + y, then g′(y) = (2y + 1) and g(θ) = W. Then by theDelta Method, the asymptotic distribution of W can be written as

W = g(θ) ∼ AN(

g(θ), g′(θ)σ2(θ)

n

)= AN

(θ2 + θ,

(2θ + 1)2σ2(θ)

n

)= AN

(θ2 + θ,

4θ3 + 2θ2

n

)σ2(θ) =

4θ3 + 2θ2

(2θ + 1)2=

2θ2(2θ + 1)

(2θ + 1)2=

2θ2

2θ + 1

Hyun Min Kang Biostatistics 602 - Lecture 26 Apil 23rd, 2013 28 / 31

Page 144: Biostatistics 602 - Statistical Inference Lecture 26 Final ... · Biostatistics 602 - Statistical Inference Lecture 26 Final Exam Review & Practice Problems for the Final Hyun Min

..........

.....

......

.....

.....

.....

......

.....

.....

.....

......

.....

.....

.....

......

.....

......

.....

.....

.

. . . . . . . .Review

. . . .P1

. . . . . .P2

. . . . .P3

. . . . . .P4

.Wrap-up

(b) - Asymptotic distribution of MLE (cont’d)

The asymptotic distribution of MLE θ

θ ∼ AN(θ,

σ2(θ)

n

)= AN

(θ,

2θ2

n(2θ + 1)

)

Note that you cannot use CR-bound for the asymptotic variance of MLEbecause the regularity condition does not hold (open set criteria).

Hyun Min Kang Biostatistics 602 - Lecture 26 Apil 23rd, 2013 29 / 31

Page 145: Biostatistics 602 - Statistical Inference Lecture 26 Final ... · Biostatistics 602 - Statistical Inference Lecture 26 Final Exam Review & Practice Problems for the Final Hyun Min

..........

.....

......

.....

.....

.....

......

.....

.....

.....

......

.....

.....

.....

......

.....

......

.....

.....

.

. . . . . . . .Review

. . . .P1

. . . . . .P2

. . . . .P3

. . . . . .P4

.Wrap-up

(b) - Asymptotic distribution of MLE (cont’d)

The asymptotic distribution of MLE θ

θ ∼ AN(θ,

σ2(θ)

n

)= AN

(θ,

2θ2

n(2θ + 1)

)Note that you cannot use CR-bound for the asymptotic variance of MLEbecause the regularity condition does not hold (open set criteria).

Hyun Min Kang Biostatistics 602 - Lecture 26 Apil 23rd, 2013 29 / 31

Page 146: Biostatistics 602 - Statistical Inference Lecture 26 Final ... · Biostatistics 602 - Statistical Inference Lecture 26 Final Exam Review & Practice Problems for the Final Hyun Min

..........

.....

......

.....

.....

.....

......

.....

.....

.....

......

.....

.....

.....

......

.....

......

.....

.....

.

. . . . . . . .Review

. . . .P1

. . . . . .P2

. . . . .P3

. . . . . .P4

.Wrap-up

(c) - ARE of MLE compared to X

By CLT, the asymptotic distribution of X is

X ∼ AN(θ,

θ

n

)

Then, ARE(θ,X) is

ARE(θ,X) =θ2θ2

2θ+1

=2θ + 1

2θ= 1 +

1

2θ> 1

Therefore, θ is more efficient estimator than X.

Hyun Min Kang Biostatistics 602 - Lecture 26 Apil 23rd, 2013 30 / 31

Page 147: Biostatistics 602 - Statistical Inference Lecture 26 Final ... · Biostatistics 602 - Statistical Inference Lecture 26 Final Exam Review & Practice Problems for the Final Hyun Min

..........

.....

......

.....

.....

.....

......

.....

.....

.....

......

.....

.....

.....

......

.....

......

.....

.....

.

. . . . . . . .Review

. . . .P1

. . . . . .P2

. . . . .P3

. . . . . .P4

.Wrap-up

(c) - ARE of MLE compared to X

By CLT, the asymptotic distribution of X is

X ∼ AN(θ,

θ

n

)Then, ARE(θ,X) is

ARE(θ,X) =θ2θ2

2θ+1

=2θ + 1

2θ= 1 +

1

2θ> 1

Therefore, θ is more efficient estimator than X.

Hyun Min Kang Biostatistics 602 - Lecture 26 Apil 23rd, 2013 30 / 31

Page 148: Biostatistics 602 - Statistical Inference Lecture 26 Final ... · Biostatistics 602 - Statistical Inference Lecture 26 Final Exam Review & Practice Problems for the Final Hyun Min

..........

.....

......

.....

.....

.....

......

.....

.....

.....

......

.....

.....

.....

......

.....

......

.....

.....

.

. . . . . . . .Review

. . . .P1

. . . . . .P2

. . . . .P3

. . . . . .P4

.Wrap-up

(c) - ARE of MLE compared to X

By CLT, the asymptotic distribution of X is

X ∼ AN(θ,

θ

n

)Then, ARE(θ,X) is

ARE(θ,X) =θ2θ2

2θ+1

=2θ + 1

2θ= 1 +

1

2θ> 1

Therefore, θ is more efficient estimator than X.

Hyun Min Kang Biostatistics 602 - Lecture 26 Apil 23rd, 2013 30 / 31

Page 149: Biostatistics 602 - Statistical Inference Lecture 26 Final ... · Biostatistics 602 - Statistical Inference Lecture 26 Final Exam Review & Practice Problems for the Final Hyun Min

..........

.....

......

.....

.....

.....

......

.....

.....

.....

......

.....

.....

.....

......

.....

......

.....

.....

.

. . . . . . . .Review

. . . .P1

. . . . . .P2

. . . . .P3

. . . . . .P4

.Wrap-up

(c) - ARE of MLE compared to X

By CLT, the asymptotic distribution of X is

X ∼ AN(θ,

θ

n

)Then, ARE(θ,X) is

ARE(θ,X) =θ2θ2

2θ+1

=2θ + 1

2θ= 1 +

1

2θ> 1

Therefore, θ is more efficient estimator than X.

Hyun Min Kang Biostatistics 602 - Lecture 26 Apil 23rd, 2013 30 / 31

Page 150: Biostatistics 602 - Statistical Inference Lecture 26 Final ... · Biostatistics 602 - Statistical Inference Lecture 26 Final Exam Review & Practice Problems for the Final Hyun Min

..........

.....

......

.....

.....

.....

......

.....

.....

.....

......

.....

.....

.....

......

.....

......

.....

.....

.

. . . . . . . .Review

. . . .P1

. . . . . .P2

. . . . .P3

. . . . . .P4

.Wrap-up

Wrapping Up

..1 Many thanks for your attentions and feedbacks.

..2 Please complete your teaching evaluations, which will be very helpfulfor further improvement in the next year.

..3 Final exam will be Thursday April 25th, 4:00-6:00pm.

..4 The last office hour will be held Wednesday April 24th, 4:00-5:00pm.

..5 The grade will be posted during the weekend.

..6 Don’t forget the materials we have learned, because they are the keytopics for your candidacy exam.

Hyun Min Kang Biostatistics 602 - Lecture 26 Apil 23rd, 2013 31 / 31

Page 151: Biostatistics 602 - Statistical Inference Lecture 26 Final ... · Biostatistics 602 - Statistical Inference Lecture 26 Final Exam Review & Practice Problems for the Final Hyun Min

..........

.....

......

.....

.....

.....

......

.....

.....

.....

......

.....

.....

.....

......

.....

......

.....

.....

.

. . . . . . . .Review

. . . .P1

. . . . . .P2

. . . . .P3

. . . . . .P4

.Wrap-up

Wrapping Up

..1 Many thanks for your attentions and feedbacks.

..2 Please complete your teaching evaluations, which will be very helpfulfor further improvement in the next year.

..3 Final exam will be Thursday April 25th, 4:00-6:00pm.

..4 The last office hour will be held Wednesday April 24th, 4:00-5:00pm.

..5 The grade will be posted during the weekend.

..6 Don’t forget the materials we have learned, because they are the keytopics for your candidacy exam.

Hyun Min Kang Biostatistics 602 - Lecture 26 Apil 23rd, 2013 31 / 31

Page 152: Biostatistics 602 - Statistical Inference Lecture 26 Final ... · Biostatistics 602 - Statistical Inference Lecture 26 Final Exam Review & Practice Problems for the Final Hyun Min

..........

.....

......

.....

.....

.....

......

.....

.....

.....

......

.....

.....

.....

......

.....

......

.....

.....

.

. . . . . . . .Review

. . . .P1

. . . . . .P2

. . . . .P3

. . . . . .P4

.Wrap-up

Wrapping Up

..1 Many thanks for your attentions and feedbacks.

..2 Please complete your teaching evaluations, which will be very helpfulfor further improvement in the next year.

..3 Final exam will be Thursday April 25th, 4:00-6:00pm.

..4 The last office hour will be held Wednesday April 24th, 4:00-5:00pm.

..5 The grade will be posted during the weekend.

..6 Don’t forget the materials we have learned, because they are the keytopics for your candidacy exam.

Hyun Min Kang Biostatistics 602 - Lecture 26 Apil 23rd, 2013 31 / 31

Page 153: Biostatistics 602 - Statistical Inference Lecture 26 Final ... · Biostatistics 602 - Statistical Inference Lecture 26 Final Exam Review & Practice Problems for the Final Hyun Min

..........

.....

......

.....

.....

.....

......

.....

.....

.....

......

.....

.....

.....

......

.....

......

.....

.....

.

. . . . . . . .Review

. . . .P1

. . . . . .P2

. . . . .P3

. . . . . .P4

.Wrap-up

Wrapping Up

..1 Many thanks for your attentions and feedbacks.

..2 Please complete your teaching evaluations, which will be very helpfulfor further improvement in the next year.

..3 Final exam will be Thursday April 25th, 4:00-6:00pm.

..4 The last office hour will be held Wednesday April 24th, 4:00-5:00pm.

..5 The grade will be posted during the weekend.

..6 Don’t forget the materials we have learned, because they are the keytopics for your candidacy exam.

Hyun Min Kang Biostatistics 602 - Lecture 26 Apil 23rd, 2013 31 / 31

Page 154: Biostatistics 602 - Statistical Inference Lecture 26 Final ... · Biostatistics 602 - Statistical Inference Lecture 26 Final Exam Review & Practice Problems for the Final Hyun Min

..........

.....

......

.....

.....

.....

......

.....

.....

.....

......

.....

.....

.....

......

.....

......

.....

.....

.

. . . . . . . .Review

. . . .P1

. . . . . .P2

. . . . .P3

. . . . . .P4

.Wrap-up

Wrapping Up

..1 Many thanks for your attentions and feedbacks.

..2 Please complete your teaching evaluations, which will be very helpfulfor further improvement in the next year.

..3 Final exam will be Thursday April 25th, 4:00-6:00pm.

..4 The last office hour will be held Wednesday April 24th, 4:00-5:00pm.

..5 The grade will be posted during the weekend.

..6 Don’t forget the materials we have learned, because they are the keytopics for your candidacy exam.

Hyun Min Kang Biostatistics 602 - Lecture 26 Apil 23rd, 2013 31 / 31

Page 155: Biostatistics 602 - Statistical Inference Lecture 26 Final ... · Biostatistics 602 - Statistical Inference Lecture 26 Final Exam Review & Practice Problems for the Final Hyun Min

..........

.....

......

.....

.....

.....

......

.....

.....

.....

......

.....

.....

.....

......

.....

......

.....

.....

.

. . . . . . . .Review

. . . .P1

. . . . . .P2

. . . . .P3

. . . . . .P4

.Wrap-up

Wrapping Up

..1 Many thanks for your attentions and feedbacks.

..2 Please complete your teaching evaluations, which will be very helpfulfor further improvement in the next year.

..3 Final exam will be Thursday April 25th, 4:00-6:00pm.

..4 The last office hour will be held Wednesday April 24th, 4:00-5:00pm.

..5 The grade will be posted during the weekend.

..6 Don’t forget the materials we have learned, because they are the keytopics for your candidacy exam.

Hyun Min Kang Biostatistics 602 - Lecture 26 Apil 23rd, 2013 31 / 31


Related Documents